[obm-l] Re: [obm-l] Re: [obm-l] Ângulos de um triângulo

2020-12-04 Por tôpico Carlos Victor
 

Use a lei dos senos e o fato de que sen(54º)-sen(18º)=sen(30º). 

Em 04/12/2020 1:50, Anderson Torres escreveu: 

> Em seg., 30 de nov. de 2020 às 19:28, Professor Vanderlei Nemitz 
>  escreveu: 
> 
>> Boa noite!
>> 
>> Alguém conhece uma saída para o seguinte problema? 
>> Muito obrigado! 
>> 
>> NUM TRIÂNGULO ISÓSCELES ABC, AB = AC. 
>> SEJA D UM PONTO INTERNO TAL QUE OS ÂNGULOS DBC, DCB, DBA E DCA MEDEM, 
>> RESPECTIVAMENTE, 12°, 18°, 54° E 48°. 
>> DETERMINE A MEDIDA DO ÂNGULO DAC.
> 
> Eu ainda nao resolvi, mas sei que e 30 graus. 
> 
>> [1]
>> Livre de vírus. www.avast.com [1].

 

Links:
--
[1]
https://www.avast.com/sig-email?utm_medium=email&utm_source=link&utm_campaign=sig-email&utm_content=webmail

Re: [obm-l] Prova interessante de que lim n ---> oo n^(1/n) = 1

2020-10-29 Por tôpico Carlos Victor
 

Muito linda Artur. 

Carlos Victor 

Em 28/10/2020 7:44, Artur Costa Steiner escreveu: 

> Achei essa prova bem imaginativa. 
> 
> Para n>= 2, temos n^(1/n) > 1. n^(1/n) pode ser escrito como 
> 
> n^(1/n) = ((raiz(n) . raiz(n) . 1  1)^(1/n) 
> 
> onde o 1 aparece n - 2 vezes. Logo, n^(1/n) é a média geométrica dos números 
> {raiz(n), raiz(n), 1, . .1}. 
> 
> Pela desigualdade MA >= MG temos, para n>= 2, que 
> 
> 1 < n^(1/n) < (raiz(n) + raiz(n) + 1 +1)/n= (2 raiz(n) + (n - 2))/n 
> 
> 1 < n^(1/n) < 2/raiz(n) + 1 - 2/n 
> 
> Como na desigualdade acima o membro da direita tende a 1 quando n vai para 
> oo, segue-se por confronto que 
> 
> lim n ---> oo n^(1/n) = 1 
> 
> Artur

 

Re: [obm-l] teste

2020-08-08 Por tôpico Carlos Victor
 

Há muito tempo que os meus emails enviados também estão assim e não sei
o motivo. 

Carlos Victor 

PS : este email não sei se chegará aos companheiros da lista 

Em 08/08/2020 17:39, Luís Lopes escreveu: 

> Recebo as mensagens normalmente. Mas não tenho confirmação de 
> chegada ao grupo das que envio. E não aparecem nos arquivos também. 
> 
> Mandei uma há umas 5 horas intitulada polinômio minimal. 
> Chegou? Alguém recebeu ? 
> 
> Luís 
> 
> -- 
> Esta mensagem foi verificada pelo sistema de antivírus e 
> acredita-se estar livre de perigo.

 
-- 
Esta mensagem foi verificada pelo sistema de antiv�rus e
 acredita-se estar livre de perigo.



Re: [obm-l] Dois problemas

2020-04-26 Por tôpico Carlos Victor
 

Para o (1), observar que a_n é periódico e tem período igual a 20, daí
 

Abraços 

Carlos Victor 

Em 26/04/2020 19:21, Rogério Possi Júnior escreveu: 

> Boa noite. 
> 
> Quem pode ajudar com esses dois problemas: 
> 
> 1) (Ibero-1992) Para cada inteiro positivo n, seja a_n o último dígito de 
> 1+2+3+...+n. Calcule a_1+a_2+...+a_n. 
> 
> 2) (UK-1997) N é um número inteiro de 4 dígitos não terminado em zero, e R(N) 
> é o número inteiro de 4 dígitos obtido pela reversão dos dígitos de N; por 
> exemplo R(3275)=5723. Determine todos os inteiros N ára os quais R(N)=4N+3. 
> 
> Sds, 
> 
> Rogério 
> 
> -- 
> Esta mensagem foi verificada pelo sistema de antivírus e 
> acredita-se estar livre de perigo.

 
-- 
Esta mensagem foi verificada pelo sistema de antiv�rus e
 acredita-se estar livre de perigo.



Re: [obm-l]

2020-04-05 Por tôpico Carlos Victor
 

Inscrevendo o triângulo em um círculo, é possível chegar a esta
resposta. 

Carlos Victor 

Em 05/04/2020 19:10, Anderson Torres escreveu: 

> Em dom., 5 de abr. de 2020 às 19:09, Anderson Torres
>  escreveu: 
> Em qui., 13 de fev. de 2020 às 18:19, Vanderlei Nemitz
>  escreveu: 
> Usei várias leis dos senos, obtive coisas legais, mas não o ângulo pedido. 
> Alguém conhece algo interessante?
> 
> Muito obrigado!
> 
> Em um triângulo ABC, em AC localiza-se os pontos consecutivos M,Q e N, tal 
> que AM=NC. Se Q é ponto médio de MN e os ângulos NBC e ABM medem 20º, calcule 
> a medida do ângulo BQC. 
> Se fizermos BAC=\alpha, BCA=\gamma, obtemos que BM/sin alpha = AM/sin
> 20 = CN/sin 20 = BN/sin gamma
> 
> Também, MNB=20+gamma e NMB = 20+alpha.

Dessa forma, usando outra lei dos senos, temos sin alpha / sin
(20+gamma) = sin gamma / sin (20+alpha).

O que nos dá cos (20+2 alpha) = cos (20 + 2 gamma), o que implica
alpha = gamma, ou ABC isósceles, portanto BQC = 90.

>> --
>> Esta mensagem foi verificada pelo sistema de antivírus e
>> acredita-se estar livre de perigo.

 
-- 
Esta mensagem foi verificada pelo sistema de antiv�rus e
 acredita-se estar livre de perigo.



Re: [obm-l] teoria dos numeros

2020-03-30 Por tôpico Carlos Victor
 

Basta fazer (2^3-1)^2n+(2^3+1)^2n -2 e usar binômio de Newton. 

Em 28/03/2020 13:55, Israel Meireles Chrisostomo escreveu: 

> Eu sei resolver o problema abaixo,porém não sei se é a forma mais simples de 
> se fazer.Vcs poderiam por favor colocar suas soluções nos comentários dessa 
> publicação? O problema é o seguinte:
> Prove que 128 divide 49^{n} + 81^{n} -2, para todo n ≥ 1.Se possível não use 
> indução, pois eu já estou usando indução. 
> 
> -- 
> 
> Israel Meireles Chrisostomo 
> -- 
> Esta mensagem foi verificada pelo sistema de antivrus e 
> acredita-se estar livre de perigo.

 
-- 
Esta mensagem foi verificada pelo sistema de antiv�rus e
 acredita-se estar livre de perigo.



[obm-l] Re: [obm-l] Ajuda com dízima

2020-02-24 Por tôpico Carlos Victor
 

Estou conjecturando que 1/3^n tem período igual a 3^(n-2) , para n>=3. 

Carlos Victor 

Em 20/02/2020 18:01, Prof. Douglas Oliveira escreveu: 

> Qual o número de dígitos do período de 1/(3^2005) ? 
> 
> Saudações 
> Douglas Oliveira 
> -- 
> Esta mensagem foi verificada pelo sistema de antivrus e 
> acredita-se estar livre de perigo.

 
-- 
Esta mensagem foi verificada pelo sistema de antiv�rus e
 acredita-se estar livre de perigo.



Re: [obm-l]

2019-11-18 Por tôpico Carlos Victor
 

O Pacini me pediu que enviasse para a lista a ideia abaixo, pois ele não
está conseguindo concluir o devido envio : 

Para n par o link que o Carlos Gustavo colocou mostra a análise. 

Acredito ter encontrado uma outra ideia para todas as soluções com
a=2n+1, usando 

3^(2n+1) = 2(b^2) + 1 

3^(2n+1) = 2(b^2) +3 -2 

3(3^(2n)-1) = 2(b^2 - 1) 

3(3^n-1)(3^n+1) = 2(b-1)(b+1). 

Vou verificar se realmente usando esta ideia chegarei às soluções e
postarei mais adiante. 

Pacini 

Carlos Victor 

Em 16/11/2019 14:47, Pedro José escreveu: 

> Boa tarde! 
> Curioso, a solução (2,2) sai para q =0 no segundo caso 3q+2. 
> Todavia, falta mostrar que para os côngruos de 3 mod81, embora 6q^2+8q+3 
> dívida 81, não é uma potência de 3, já vi que ficou capenga. 
> Saudações, 
> PJMS 
> 
> Em sáb, 16 de nov de 2019 14:54, Pedro José  escreveu: 
> 
>> Boa tarde! 
>> O Esdras conseguiu para a e b par. 
>> Creio ter conseguido para a e b ímpares. 
>> Já havia encontrado (1,1) é (5,11)além de (2,2) para se b pares. 
>> Vamos atrás dos peixes maiores. 
>> 3^a=2*(3q+c)^2+1, 0=> c=1 ou c=2. 
>> Para c=1. 
>> 3^a=18q^2+12q+3 
>> 3^(a-1)=6q^2+4q+1 
>> Note que a solução (1,1) acontece para q =0. 
>> E novamente uma restrição q=2 mod3pois já encontramos a soluçao para q =0. 
>> Mas como não havia soluções menores que as citadas com a e b ímpares, 
>> 3^a>81. 
>> Então 81 |6q^2+4q+1 Para algum resíduo de{5, 8 , 11...77,80}, o que não 
>> acontece. 
>> Para c =2 
>> 3^a =2(3q^2+2)^2+1 
>> 3^(a-1)=6q^2+8q+3 E temos nova restrição q=0 mod3. 
>> Observe que a solução (5,11) vem de q=3. 
>> Usando o mesmo raciocínio anterior, 
>> 81 | 6q^2+8q+3 para algum resíduo de {6,9,12..75 78} 
>> O que não acontece. 
>> Então juntando essa restrição braçal com a refinada do Esdras só existem as 
>> soluções que mencionara lá atrás.: (1,1); (2,2) e (5,11). 
>> Alguém poderia verificar se está correto? 
>> Saudações, 
>> PJMS
> 
> -- 
> Esta mensagem foi verificada pelo sistema de antivrus e 
> acredita-se estar livre de perigo.

 
-- 
Esta mensagem foi verificada pelo sistema de antiv�rus e
 acredita-se estar livre de perigo.



Re: [obm-l]

2019-11-18 Por tôpico Carlos Victor
 

O Pacini me pediu que enviasse para a lista a ideia abaixo, pois ele não
está conseguindo concluir o devido envio : 

Para n par o link que o Carlos Gustavo colocou mostra a análise. 

Acredito ter encontrado uma outra ideia para todas as soluções com
a=2n+1, usando 

3^(2n+1) = 2(b^2) + 1 

3^(2n+1) = 2(b^2) +3 -2 

3(3^(2n)-1) = 2(b^2 - 1) 

3(3^n-1)(3^n+1) = 2(b-1)(b+1). 

Vou verificar se realmente usando esta ideia chegarei às soluções e
postarei mais adiante. 

Pacini 

Carlos Victor 

Em 16/11/2019 14:47, Pedro José escreveu: 

> Boa tarde! 
> Curioso, a solução (2,2) sai para q =0 no segundo caso 3q+2. 
> Todavia, falta mostrar que para os côngruos de 3 mod81, embora 6q^2+8q+3 
> dívida 81, não é uma potência de 3, já vi que ficou capenga. 
> Saudações, 
> PJMS 
> 
> Em sáb, 16 de nov de 2019 14:54, Pedro José  escreveu: 
> 
>> Boa tarde! 
>> O Esdras conseguiu para a e b par. 
>> Creio ter conseguido para a e b ímpares. 
>> Já havia encontrado (1,1) é (5,11)além de (2,2) para se b pares. 
>> Vamos atrás dos peixes maiores. 
>> 3^a=2*(3q+c)^2+1, 0=> c=1 ou c=2. 
>> Para c=1. 
>> 3^a=18q^2+12q+3 
>> 3^(a-1)=6q^2+4q+1 
>> Note que a solução (1,1) acontece para q =0. 
>> E novamente uma restrição q=2 mod3pois já encontramos a soluçao para q =0. 
>> Mas como não havia soluções menores que as citadas com a e b ímpares, 
>> 3^a>81. 
>> Então 81 |6q^2+4q+1 Para algum resíduo de{5, 8 , 11...77,80}, o que não 
>> acontece. 
>> Para c =2 
>> 3^a =2(3q^2+2)^2+1 
>> 3^(a-1)=6q^2+8q+3 E temos nova restrição q=0 mod3. 
>> Observe que a solução (5,11) vem de q=3. 
>> Usando o mesmo raciocínio anterior, 
>> 81 | 6q^2+8q+3 para algum resíduo de {6,9,12..75 78} 
>> O que não acontece. 
>> Então juntando essa restrição braçal com a refinada do Esdras só existem as 
>> soluções que mencionara lá atrás.: (1,1); (2,2) e (5,11). 
>> Alguém poderia verificar se está correto? 
>> Saudações, 
>> PJMS
> 
> -- 
> Esta mensagem foi verificada pelo sistema de antivrus e 
> acredita-se estar livre de perigo.

 
-- 
Esta mensagem foi verificada pelo sistema de antiv�rus e
 acredita-se estar livre de perigo.



Re: [obm-l]

2019-11-18 Por tôpico Carlos Victor
 

O Pacini me pediu que enviasse para a lista a ideia abaixo, pois ele não
está conseguindo concluir o devido envio : 

Para n par o link que o Carlos Gustavo colocou mostra a análise. 

Acredito ter encontrado uma outra ideia para todas as soluções com
a=2n+1, usando 

3^(2n+1) = 2(b^2) + 1 

3^(2n+1) = 2(b^2) +3 -2 

3(3^(2n)-1) = 2(b^2 - 1) 

3(3^n-1)(3^n+1) = 2(b-1)(b+1). 

Vou verificar se realmente usando esta ideia chegarei às soluções e
postarei mais adiante. 

Pacini 

Carlos Victor 

Em 12/11/2019 19:06, Carlos Gustavo Tamm de Araujo Moreira escreveu: 

> Há uma menção a esse problema em 
> https://math.stackexchange.com/questions/2826307/integer-solutions-of-3n-1-2m2
>  [1] 
> Uma sugestão é usar o fato de que Z[i.sqrt(2)] é um domínio de fatoração 
> única, e escrever 1+2b^2 como (1+b.i.sqrt(2))(1-b.i.sqrt(2)). 
> Notem que 3 se fatora aí como (1+i.sqrt(2))(1- i.sqrt(2)). 
> Abraços, 
> Gugu 
> 
> On Tue, Nov 12, 2019 at 7:21 PM Pedro José  wrote: 
> 
> Boa noite! 
> Agora captei vosso pensamento. 
> Só que ao transformar a equação em uma equação de Pell, nós maculamos a 
> função 3^n. 
> Em verdade a solução para a par a= 2n, seria (2,2); pois, como mencionara 
> anteriormente se a é par, b também o é. 
> Só que quando procuramos as outras soluções, baseando-se na propriedade de 
> que a norma em Q [RAiz(A)] conserva a multiplicação. Só que quando eu pego a 
> solução 
> 3 + 2 Raiz(2) e elevo ao quadrado 17 + 12 Raiz(2). Se eu pegar 17^2-2*12^2=1 
> eu atendo x^2 - 2Y^2=1. E assim sucessivamente. Mas não existe n inteiro tal 
> que 3^n=17, então não é uma solução da equação original. 
> Creio que seja um pouco mais complicada a solução. Pois o difícil é saber 
> quando atende também a 3^n. 
> Acredito que deva haver uma forma de restringir a essas soluções, pois, 
> definir em que condições a solução terá x como uma potência de 3 seja bem 
> difícil. 
> Estou apanhando mais do que mala velha em véspera de viagem. 
> Se alguém postar uma solução, me ajudaria bastante. 
> 
> Saudações, 
> PJMS 
> 
> Saudações, 
> PJMS. 
> 
> Em ter., 12 de nov. de 2019 às 17:25, Pedro José  
> escreveu: 
> 
> Boa tarde! 
> Douglas, 
> perdoe-me pela minha miopia, mas você poderia detalhar melhor onde entra a 
> equação de Pell? 
> A equação de Pell não é x^2-Dy^2 = N? 
> Se a é par b é par e se a ímpar b é ímpar para atender mod8, 
> Não consegui captar a sugestão. 
> 
> Saudações, 
> PJMS 
> 
> Em ter., 12 de nov. de 2019 às 16:50, Prof. Douglas Oliveira 
>  escreveu: 
> Hum, então, vamos analisar o caso de a ser par do tipo 2n. 
> 
> Assim podemos escrever que (3^n+b(sqrt2))(3^n-b(sqrt2))=1 
> Dai através da solução mínima que o Pedro fez, como (1,1) por exemplo, da pra 
> ver que são infinitas soluções usando a equação de Pell. 
> 
> Abraco 
> Douglas Oliveira. 
> 
> Em dom, 10 de nov de 2019 19:33, gilberto azevedo  
> escreveu: 
> 
> [HELP] 
> Achas todos os pares (a,b) inteiros positivos tais que : 
> 3^a = 2b² + 1. 
> 
> -- 
> Esta mensagem foi verificada pelo sistema de antivírus e 
> acredita-se estar livre de perigo. 
> -- 
> Esta mensagem foi verificada pelo sistema de antivírus e 
> acredita-se estar livre de perigo.

-- 
Esta mensagem foi verificada pelo sistema de antivírus e 
acredita-se estar livre de perigo. 
-- 
Esta mensagem foi verificada pelo sistema de antivrus e 
acredita-se estar livre de perigo. 
 

Links:
--
[1]
https://math.stackexchange.com/questions/2826307/integer-solutions-of-3n-1-2m2
-- 
Esta mensagem foi verificada pelo sistema de antiv�rus e
 acredita-se estar livre de perigo.



[obm-l] Re: [obm-l] Re: [obm-l] Re: [obm-l] Re: [obm-l] Demonstração com Geometria Plana?

2018-11-24 Por tôpico Carlos Victor
 

Oi Vanderlei, vamos lá: 

Seja ABCD o quadrado de diagonais AC e BD. Sejam os pontos P, E e F como
no enunciado. Tracemos a reta que passa por A e E encontrando o
prolongamento de DC em R.Seja também Q o ponto de interseção da reta que
passa por B e F com o prolongamento de DC.Seja T a interseção da reta
que passa por Q e E com o lado AB. 

Sejam BP=z, quadrado de lado AB=L, TB=k, CQ=x e QR=y. Por semelhança de
triângulos verifique que : 

x/k =L/z e y/L =x/z donde x^2=ky. Agora por semelhança veja que 

y/AT= x/k ou seja ky=x.AT e como ky=x^2 temos que x=AT ou seja CQ=AT. 

Como CQ é paralelo a AT e congruentes, temos que o quadrilátero ACQT é
um paralelogramo e já que as diagonais do quadrado são perpendiculares
temos que QT é perpendicular a BD. 

Temos então que no triângulo BDQ, BC e QH( H é a interseção de QT com
BD); 

ou seja E é o ortocentro de BDQ; donde PD é perpendicular a BQ. 

Verifiquem se há algum erro, ok? 

Abraços 

Carlos Victor 

Em 23/11/2018 22:38, Vanderlei Nemitz escreveu: 

> Estamos aguardando o Carlos Victor... 
> :) 
> 
> Em sex, 23 de nov de 2018 18:14, Mauricio de Araujo 
>  Alguem conseguiu finalizar a demonstração? 
> 
> Em qua, 21 de nov de 2018 11:52, Vanderlei Nemitz  escreveu: 
> Hummm... 
> Parece que prolongando BF e DC, que se encontram num ponto Q, E é o 
> ortocentro do triângulo BDQ. 
> O desenho sugere isso. 
> Mas como mostrar isso? 
> 
> Em ter, 20 de nov de 2018 23:38, Carlos Victor  escreveu: 
> 
> Oi Vanderlei, 
> 
> Uma dica : tente mostrar que o ponto E é o ortocentro de um triângulo " 
> estratégico". É muito legal que você descubra sozinho 
> 
> Abraços 
> 
> Carlos Victor 
> 
> Em 20/11/2018 17:33, Vanderlei Nemitz escreveu: 
> Pessoal, o seguinte problema sai "tranquilamente" usando Geometria Analítica. 
> Tentei usar Geometria Plana, mas apenas girei bastante, sem concluir. Será 
> que é possível? 
> 
> Dado um ponto P situado no prolongamento do lado AB de um quadrado ABCD, 
> traçam-se as retas PC e PD. Pelo ponto E, intersecção de BC e PD, conduzimos 
> a reta AE cuja intersecção com PC é o ponto F. Provar que BF e PD são 
> perpendiculares. 
> -- 
> Esta mensagem foi verificada pelo sistema de antivrus e 
> acredita-se estar livre de perigo.

-- 
Esta mensagem foi verificada pelo sistema de antivírus e 
acredita-se estar livre de perigo. 
-- 
Esta mensagem foi verificada pelo sistema de antivírus e 
acredita-se estar livre de perigo. 
-- 
Esta mensagem foi verificada pelo sistema de antivrus e 
acredita-se estar livre de perigo. 
 
-- 
Esta mensagem foi verificada pelo sistema de antiv�rus e
 acredita-se estar livre de perigo.



[obm-l] Re: [obm-l] Re: [obm-l] Re: [obm-l] Demonstração com Geometria Plana?

2018-11-23 Por tôpico Carlos Victor
 

Desculpem, estou em trânsito. Até amanhã eu posto, ok ? 

Abraços 

Em 23/11/2018 18:05, Mauricio de Araujo escreveu: 

> Alguem conseguiu finalizar a demonstração? 
> 
> Em qua, 21 de nov de 2018 11:52, Vanderlei Nemitz  escreveu: 
> Hummm... 
> Parece que prolongando BF e DC, que se encontram num ponto Q, E é o 
> ortocentro do triângulo BDQ. 
> O desenho sugere isso. 
> Mas como mostrar isso? 
> 
> Em ter, 20 de nov de 2018 23:38, Carlos Victor  escreveu: 
> 
> Oi Vanderlei, 
> 
> Uma dica : tente mostrar que o ponto E é o ortocentro de um triângulo " 
> estratégico". É muito legal que você descubra sozinho 
> 
> Abraços 
> 
> Carlos Victor 
> 
> Em 20/11/2018 17:33, Vanderlei Nemitz escreveu: 
> Pessoal, o seguinte problema sai "tranquilamente" usando Geometria Analítica. 
> Tentei usar Geometria Plana, mas apenas girei bastante, sem concluir. Será 
> que é possível? 
> 
> Dado um ponto P situado no prolongamento do lado AB de um quadrado ABCD, 
> traçam-se as retas PC e PD. Pelo ponto E, intersecção de BC e PD, conduzimos 
> a reta AE cuja intersecção com PC é o ponto F. Provar que BF e PD são 
> perpendiculares. 
> -- 
> Esta mensagem foi verificada pelo sistema de antivrus e 
> acredita-se estar livre de perigo.

-- 
Esta mensagem foi verificada pelo sistema de antivírus e 
acredita-se estar livre de perigo. 
-- 
Esta mensagem foi verificada pelo sistema de antivrus e 
acredita-se estar livre de perigo. 
 
-- 
Esta mensagem foi verificada pelo sistema de antiv�rus e
 acredita-se estar livre de perigo.



[obm-l] Re: [obm-l] Demonstração com Geometria Plana?

2018-11-20 Por tôpico Carlos Victor
 

Oi Vanderlei, 

Uma dica : tente mostrar que o ponto E é o ortocentro de um triângulo "
estratégico". É muito legal que você descubra sozinho 

Abraços 

Carlos Victor 

Em 20/11/2018 17:33, Vanderlei Nemitz escreveu: 

> Pessoal, o seguinte problema sai "tranquilamente" usando Geometria Analítica. 
> Tentei usar Geometria Plana, mas apenas girei bastante, sem concluir. Será 
> que é possível? 
> 
> Dado um ponto P situado no prolongamento do lado AB de um quadrado ABCD, 
> traçam-se as retas PC e PD. Pelo ponto E, intersecção de BC e PD, conduzimos 
> a reta AE cuja intersecção com PC é o ponto F. Provar que BF e PD são 
> perpendiculares. 
> -- 
> Esta mensagem foi verificada pelo sistema de antivrus e 
> acredita-se estar livre de perigo.

 
-- 
Esta mensagem foi verificada pelo sistema de antiv�rus e
 acredita-se estar livre de perigo.



[obm-l] Re: [obm-l] Valor mínimo

2018-09-08 Por tôpico Carlos Victor
 

Olá Daniel, 

Esta questão saiu da original que foi de uma Olimpíada de Leningrad em
1988 cujo enunciado era : 

a,b,c e d reais positivos; prove que 1/a+1/b+4/c+16/d >= 64/(a+b+c+d). 

Tome (1/a+1/b+4/c+16/d).(a+b+c+d)= 22+(a/b+b/a)+2(2a/c+
c/2a)+4(4a/d+d/4a)+2(2b/c+c/2b)+4(4b/d+d/4b)+8(2c/d+d/2c)>=22+2+2.2+4.2+2.2+4.2+8.2=64


Abraços 

Carlos Victor 

Em 08/09/2018 9:31, Daniel Quevedo escreveu: 

> Se A, B, C e D são reais positivos então o valor mínimo de 1/A + 1/B + 4/C + 
> 16/D é igual a: 
> A) 1/(A + B +C+D) 
> B) 16/(A + B +C+D) 
> C) 2/(A + B +C+D) 
> D) 64/(A + B +C+D) 
> E) 4/(A + B +C+D) 
> 
> R: d -- 
> 
> Fiscal: Daniel Quevedo 
> -- 
> Esta mensagem foi verificada pelo sistema de antivrus e 
> acredita-se estar livre de perigo.

 
-- 
Esta mensagem foi verificada pelo sistema de antiv�rus e
 acredita-se estar livre de perigo.



[obm-l] Re: [obm-l] Equação 4 grau

2018-06-26 Por tôpico Carlos Victor
 

Oi daniel, 

Faça (x^2+1)^2 =2(x+1)^2 e . 

Abraçõs 

Carlos Victor 

Em 26/06/2018 15:09, Daniel Quevedo escreveu: 

> As raizes reais da equação x^4 -4x=1 pertencem ao intervalo: 
> A) (1,11) 
> B) (2, 12) 
> C) (3, 13) 
> D) (4, 14) 
> E) ( 5, 15) 
> 
> R: c -- 
> 
> Fiscal: Daniel Quevedo 
> -- 
> Esta mensagem foi verificada pelo sistema de antivrus e 
> acredita-se estar livre de perigo.

 
-- 
Esta mensagem foi verificada pelo sistema de antiv�rus e
 acredita-se estar livre de perigo.



[obm-l] Re: [obm-l] Re: [obm-l] Re: [obm-l] Re: [obm-l] Equação Funcional

2018-06-12 Por tôpico Carlos Victor
 

Olá pessoal, 

Devemos ser cuidadosos com este livro. Há muitas respostas
inconsistentes no gabarito. 

Carlos Victor 

Em 12/06/2018 14:00, Pedro José escreveu: 

> Boa tarde!
> 
> Acho estranho, pois fui compondo g(x) com g(x), sendo g(x)=(1-x)/x e, 
> verifiquei que nunca vai dar a identidade. Dá o quociente de duas funções 
> afins e portanto nunca dará x. Por curiosidade, os coeficientes dos 
> polinômios de primeiro grau são, em módulo, termos da sequência de Fibonacci. 
> E continuo achando estranho pois se supormos que f(x)=x^3-x^2-1/[x(x-1)] é a 
> solução temos:
> 
> f(x)+f((1-x)/x)=f(x)+f(1/x-1)= x^3-x^2-1/[x(x-1)] + 1/x^3 - 4/x^2 + 5/x - 2 + 
> x^2/(2x^2-3x+1)
> 
> só que pela definição tem que ser igual a 1+x, não vejo como será cancelado 
> esse termo em x^3, por exemplo. Será que fiz barbeiragem nesse 
> desenvolvimento?
> 
> Pois, me parece que algo de errado, ou com o enunciado ou com a solução.
> 
> Não satisfeito, peguei x = (-1+ raiz(5))/2, que é uma das raízes de (1-x)/x=x
> 
> Pela definição temos 2f((-1+raiz(5))/2) = 1 + (-1+raiz(5)/2) ==> 
> f((-1+raiz(5))/2) = (1+raiz(5))/4
> 
> mas aplicando a solução proposta:
> 
> f((-1+raiz(5)/2)) = 1/8 (-1 + 3raiz(5) -15 +raiz(5)^3) - 1/4(1 -2raiz(5) +5) 
> - 1 = 1/8 ( -12+raiz(5)+raiz(5)^3 <> (1+raiz(5))/4, que já poderia ser visto, 
> de cara, pela existência do termo raiz(5)^3.
> 
> O problema não está fechando, creio eu. 
> Ou defeito na proposição ou no resultado. 
> Saudações, PJMS
> 
> Em 11 de junho de 2018 23:31, Jeferson Almir  
> escreveu:
> 
> Esse é o problema 2901 do livro Problemas Selecionados de Matemática ( Gandhi 
> ) 
> E resposta que ele diz é 
> R: x^3 - x^2 - 1 / x(x-1) 
> 
> Em seg, 11 de jun de 2018 às 12:15, Jeferson Almir  
> escreveu: 
> 
> Isso mesmo Ralph eu sei fazer g(x) = (x-1)/x 
> 
> Em seg, 11 de jun de 2018 às 11:33, Ralph Teixeira  
> escreveu: 
> Puxa, se fosse g(x)=(x-1)/x ali dentro do segundo termo, eu sabia fazer 
> rápido... :( Era só escrever y=g(x), z=g(y), e então: 
> f(x)+f(y)=1+x 
> f(y)+f(z)=1+y 
> f(z)+f(x)=1+z 
> pois é fácil ver que g(z)=g(g(g(x)))=x. Resolvendo esse sisteminha, 
> acharíamos f(x). 
> 
> Porém, com esse enunciado... Hm, alguém confere aqui o raciocínio abaixo, 
> porque acho que eu consigo mostrar que **não dá** para resolver isso, mas 
> estou morrendo de sono, então provavelmente escrevi alguma bobagem imensa. 
> 
> Observe que g(x)=(1-x)/x é injetiva (e sua inversa é g^(-1)(y)=1/(1+y)). Dado 
> um x_0=a, crie a sequência {x_k} com k inteiro onde x_(k+1)=g(x_k) -- observe 
> que crio isto incluindo k negativo, o que é possível desde que nenhum dos 
> números da órbita seja 0 ou -1. Vou chamar o **conjunto** de valores {x_k} de 
> "órbita" do número a. 
> 
> Pois bem, a equação funcional só dá informações sobre os valores de f dentro 
> de cada órbita! Ela diz que f(x_k)+f(x_(k+1))=1+x_k (*), e mais nada, ou 
> seja, ela não relaciona os valores de f em órbitas distintas! Se a órbita é 
> infinita, isto é, se os x_k são todos distintos, você pode ESCOLHER f(a) como 
> quiser e calcular os outros f(x_k) usando (*) como recorrência. 
> 
> Agora você me pergunta: porque a órbita não fecha? Bom, você tem razão, para 
> vários valores de "a" a órbita fecha, isto é, poderia ser x_P=x_0=a para 
> algum P<>0... Mas a equação x_P=a quer dizer g(g(g(...g(a))...)=a, que é uma 
> equação quadrática (né?), e portanto tem no máximo 2 raízes reais. Então, 
> mesmo que consideremos todos os P possíveis, o conjunto dos a que fazem a 
> órbita fechar é enumerável... Bom, os reais não são enumeráveis, então há 
> várias órbitas infinitas Acho. 
> 
> Abraço, Ralph. 
> 
> P.S.: Se eu tivesse bom senso, conferia isso antes de mandar para a lista... 
> Ah, dane-se, mesmo que eu esteja errado este tipo de raciocínio é 
> interessante, não? 
> P.S.2: Se o enunciado falar que f é *contínua*, aí talvez dê para fazer algo 
> usando o limite de x_k... 
> 
> On Mon, Jun 11, 2018 at 8:32 AM Jeferson Almir  
> wrote: 
> 
> Seja f(x) uma função real definida em R -{0,1} tal que 
> 
> f(x) + f( 1-x | x ) =1 + x determine f (x) . 
> 
> Obs: ( 1-x | x) é 1-x dividido por x . 
> -- 
> Esta mensagem foi verificada pelo sistema de antivírus e 
> acredita-se estar livre de perigo. 
> -- 
> Esta mensagem foi verificada pelo sistema de antivírus e 
> acredita-se estar livre de perigo.

-- 
Esta mensagem foi verificada pelo sistema de antivírus e 
acredita-se estar livre de perigo. 
-- 
Esta mensagem foi verificada pelo sistema de antivrus e 
acredita-se estar livre de perigo. 
 
-- 
Esta mensagem foi verificada pelo sistema de antiv�rus e
 acredita-se estar livre de perigo.



Re: [obm-l] Limite

2018-03-19 Por tôpico Carlos Victor
 

Oi Vanderlei, 

Use a equivalência de Stirling : 

n! ~ n^n.e^(-n).sqrt(2pi.n) e que lim(n^(1/n)=1 e o resultado será 1/e. 

Abraços 

Carlos Victor 

Em 19/03/2018 12:27, Vanderlei Nemitz escreveu: 

> Bom dia! 
> Eu resolvi o limite a seguir de um modo muito complicado e encontrei 1/e. 
> 
> Alguém conhece alguma solução? 
> 
> lim [n!/n^n]^(1/n), quando n tende ao infinito. 
> 
> Muito obrigado! 
> -- 
> Esta mensagem foi verificada pelo sistema de antivrus e 
> acredita-se estar livre de perigo.

 
-- 
Esta mensagem foi verificada pelo sistema de antiv�rus e
 acredita-se estar livre de perigo.



Re: [obm-l] Algebra (Polinomios)

2017-07-10 Por tôpico Carlos Victor
 

Oi Douglas, faça o seguinte: 

p(x) = (x^2+x+1)^40 = [x(x+1)+1]^40 e tomando y = x(x+1) e desenvolva o
binômo de Newton 

(y+1)^40 = [y+1)^39](y+1). Observe que os três últimos do
desenvolvimento dentro dos colchetes serão : 741y^2+39y+1, pois os
anteriores serão divisíveis por (x+1)^3. 

Basta então encontrar o resto de (741y^2+39y+1)(y+1) por (x+1)^3. 

Seja g(y) = (741y^2+39y+1)(y+1) com y = x(x+1). Como estamos dividindo
por x^3+3x^2+3x+1, basta substituirmos x^3 por -3x^2-3x-1 no
desenvolvimento de g(y). 

Fazendo algumas continhas (confira), encontramos o resto igual a
820x^2+1600x+781. 

Abraços 

Carlos Victor 

Em 10/07/2017 20:37, Douglas Oliveira de Lima escreveu: 

> Encontrar o resto da divisão do polinomio (x^2+x+1)^40 por (x+1)^3. 
> 
> Obs: Sem usar derivadas. 
> 
> Douglas Oliveira. 
> -- 
> Esta mensagem foi verificada pelo sistema de antivrus e 
> acredita-se estar livre de perigo.

 
-- 
Esta mensagem foi verificada pelo sistema de antiv�rus e
 acredita-se estar livre de perigo.



[obm-l] Re: [obm-l] Polinômios

2017-05-27 Por tôpico Carlos Victor
 

Oi Wanderlei, 

seja o resto dado por R(x)=ax^3+bx^2+cx+d. 

Onde tiver x^2 em R(x) substitua por (-x-1) e force ser igual a -x+1;
encontrando : 

c-b=-1 e a+d-b=1. Depois onde tiver x^2 substitua por(x-1) e force ser
igual a 3x+5; encontrando b+c=3 e d-b-a=5. 

conclusão : a=-2, b=2 , c=1 e d=5. 

Abraços 

Carlos Victor 

Em 27/05/2017 11:17, Vanderlei Nemitz escreveu: 

> Bom dia! 
> 
> Alguém poderia dar uma ideia na seguinte questão? Já tentes algumas 
> estratégias, mas sem êxito. 
> 
> UM POLINÔMIO P(X) DIVIDIDO POR X^2 + X + 1 DÁ RESTO -X + 1 E DIVIDIDO POR X^2 
> -X + 1 DÁ RESTO 3X + 5. QUAL O RESTO DA DIVISÃO DE P(X) POR X^4 + X^2 + 1? 
> 
> A resposta que tenho é -2X^3 + 2X^2 + X + 5. 
> 
> Obrigado! 
> 
> Vanderlei 
> -- 
> Esta mensagem foi verificada pelo sistema de antivrus e 
> acredita-se estar livre de perigo.

 
-- 
Esta mensagem foi verificada pelo sistema de antiv�rus e
 acredita-se estar livre de perigo.



[obm-l] Re: [obm-l] [obm-l] Questão Geometria

2016-10-10 Por tôpico Carlos Victor
 

Olá Vinicius, 

Seja R a intersecção de AO com BC. Seja T a intersecção da bissetriz de
 Será que alguém poria me ajudar na seguinte questão? 
> 
> * 
> 
> (Belarus) Seja O o centro do círculo ex-inscrito do triângulo ABC oposto ao 
> vértice A. Seja M o ponto médio de AC e seja P a intersec ̧ão das retas MO e 
> BC. Prove que se ∠BAC = 2∠ACB, então AB = BP. 
> 
> -- 
> Esta mensagem foi verificada pelo sistema de antivrus e 
> acredita-se estar livre de perigo.
 
-- 
Esta mensagem foi verificada pelo sistema de antiv�rus e
 acredita-se estar livre de perigo.



Re: [obm-l] Re: Geometria

2016-09-11 Por tôpico Carlos Victor
 

Oi Jeferson, 

Tome E sobre BD tal que o ângulo EAB seja 30º. Observe que o ângulo ADB
é igual a 100º e que o ângulo DAE é igual a 20º. Daí o ângulo AED é
igual a 60º. Como E está na bissetriz de ACB, então o ângulo AEC é igual
a 120º. Observe agora que D é o ponto de encontro das bissetrizes
internas do triângulo AEC e consequentemente o ângulo BDC é igual a
110º. 

Abraços 

Carlos Victor 

Em 10/09/2016 17:34, Jeferson Almir escreveu: 

> Olá pessoa queria uma ajuda nessa questão
> 
> A figura em anexo mostra um triângulo _ABC_. _D_ é um ponto interior onde a 
> medida dos ângulos _CAD_, _ABD_, _CBD_, e _BAD_ são 20º, 30º, 40º e 50º , 
> respectivamente. Encontre a medida do ângulo _BDC_. 
> 
> Em 28 de agosto de 2016 18:31, Jeferson Almir  
> escreveu:
> 
>> Olá pessoa queria uma ajuda nessa questão
>> 
>> A figura em anexo mostra um triângulo _ABC_. _D_ é um ponto interior onde a 
>> medida dos ângulos _CAD_, _ABD_, _CBD_, e _BAD_ são 20º, 30º, 40º e 50º , 
>> respectivamente. Encontre a medida do ângulo _BDC_.
> 
> -- 
> Esta mensagem foi verificada pelo sistema de antivrus e 
> acredita-se estar livre de perigo.
 
-- 
Esta mensagem foi verificada pelo sistema de antiv�rus e
 acredita-se estar livre de perigo.



[obm-l] Re: [obm-l] Re: [obm-l] Dúvida sobre a Obm U

2016-07-25 Por tôpico Carlos Victor
 

Oi Otávio, 

Você já viu a Revista Matemática Universitária da SBM ? 

Em 25/07/2016 10:09, Otávio Araújo escreveu: 

> Pois é, se algum professor com experiência em olimpíadas, como o Nicolau por 
> exemplo, respondesse minha pergunta seria de grande ajuda 
> 
> Em 24 de jul de 2016, às 23:25, Israel Meireles Chrisostomo 
>  escreveu:
> 
> Boa pergunta, eu tambÃ(c)m tenho interesse em participar da OBM U e gostaria 
> de umas dicas 
> 
> Em 16 de julho de 2016 13:29, Otávio Araújo  
> escreveu:
> Galera, gostaria que vocês me dessem dicas de o que estudar, como estudar 
> e por quais livros e materiais estudar para a prova da Obm nível 
> universitário...
> Estou muito interessado em participar, mas fico meio confuso por onde 
> estudar...
> Por favor me ajudem
> --
> Esta mensagem foi verificada pelo sistema de antivírus e
> Â acredita-se estar livre de perigo.
> 
> =
> Instruções para entrar na lista, sair da lista e usar a lista em
> http://www.mat.puc-rio.br/~obmlistas/obm-l.html [1]
> =
> 
> -- 
> Esta mensagem foi verificada pelo sistema de antivírus e 
> acredita-se estar livre de perigo.

-- 
Esta mensagem foi verificada pelo sistema de antivrus e 
acredita-se estar livre de perigo. 

Links:
--
[1] http://www.mat.puc-rio.br/~obmlistas/obm-l.html

-- 
Esta mensagem foi verificada pelo sistema de antiv�rus e
 acredita-se estar livre de perigo.



[obm-l] Re: [obm-l] [OFF] Aneis Adélicos (Adèles)

2016-05-24 Por tôpico Carlos Victor
 

OI Listeiro.
Dê uma olhada neste
material:http://www.pg.im.ufrj.br/teses/Matematica/Mestrado/319.pdf [1] 

Abraços 

Carlos Victor 

Em 23/05/2016 12:04, Listeiro 037 escreveu: 

> Saudações a todos.
> 
> Esbarrei com um conceito algébrico chamado Adele. Não encontrei
> material claro sobre este conceito. Alguém conhece algum? Desde já
> agradeço.
 

Links:
--
[1] http://www.pg.im.ufrj.br/teses/Matematica/Mestrado/319.pdf

-- 
Esta mensagem foi verificada pelo sistema de antiv�rus e
 acredita-se estar livre de perigo.



Re: [obm-l] Combinatoria

2016-02-01 Por tôpico Carlos Victor
 

Olá, 

A figura(cartela) é um retângulo dividido em seis quadrados, tendo dois
quadrados por coluna. 

Pacini 

Em 01/02/2016 3:06, Israel Meireles Chrisostomo escreveu:

Em 31 de janeiro de 2016 22:43, Pacini Bores 
escreveu:

> Olá pessoal, 
> 
> Creio que a figura não apareceu. É um retângulo dividido em seis quadrados, 
> tendo dois quadrados por coluna. 
> 
> Obrigado 
> 
> Pacini 
> 
> Em 31/01/2016 14:30, Pacini Bores escreveu: 
> 
>> Olá pessoal , poderia me ajudar na questão abaixo ? 
>> 
>> Cada cartela de uma coleção é formado por seis quadrados colorodos, 
>> justapostos como indica a figura abaixo: 
>> 
>> Em cada cartela, dois quadrados foram coloridos de azul, dois de verde e 
>> dois de rosa. A coleção apresenta as possibilidades de distribuição dessas 
>> cores nas cartelas nas condições citadas e não existem cartelas com a mesma 
>> distribuição de cores. Retirando-se ao acaso uma cartela, determine a 
>> probabilidade de que somente uma coluna apresente os quadrados de mesma cor. 
>> 
>> Agradeço desde já qualquer comentário 
>> 
>> Pacini
 

[obm-l] Re: [obm-l] Máximo absoluto de f(x) =( 5x -1) / (x^2 + 1)

2015-12-14 Por tôpico Carlos Victor
 

Oi Pedro, observe inicialmente que o campo de definição é o conjunto dos
reais. 

Chame y = (5x-1)/(x^2+1) e monte uma equação do segundo grau em x. Faça
o delta maior do que ou igual a zero. 

Abraços 

Carlos Victor 

Em 13/12/2015 22:07, Pedro Chaves escreveu: 

> Caros Colegas,
> 
> Como provar, sem recorrer a limites nem a derivadas, que existe o máximo 
> absoluto da função f(x) = (5x - 1) / (x^2 + 1), definida para todo x real?
> 
> Abraços do Pedro Chaves
> --- 
> -- 
> Esta mensagem foi verificada pelo sistema de antivírus e 
> acredita-se estar livre de perigo.
 
-- 
Esta mensagem foi verificada pelo sistema de antiv�rus e
 acredita-se estar livre de perigo.



[obm-l] Re: [obm-l] Soma de números compostos

2015-12-13 Por tôpico Carlos Victor
 

Oi Marcone, 

Para n maior do que ou igual a 1, temos: 

i)11+3n = 8+3(n+1) 

ii)11+3n+1 = 9+3(n+1) 

iii) 11+3n+2 = 10+3(n+1) 

Faltando : 12 =8+4 e 13 = 9+4. 

Abraços 

Carlos Victor 

Em 11/12/2015 23:36, marcone augusto araújo borges escreveu: 

> Mostre que todo inteiro n > 11 pode ser escrito como soma de números 
> compostos positivos 
> 
> para n par : n = 11 + 2t-1 = 4 + [2(t + 3)] 
> mas... 
> -- 
> Esta mensagem foi verificada pelo sistema de antivírus e 
> acredita-se estar livre de perigo.
 
-- 
Esta mensagem foi verificada pelo sistema de antiv�rus e
 acredita-se estar livre de perigo.



Re: [obm-l] Primo?

2015-11-25 Por tôpico Carlos Victor
 

Não. 

Observe um dos emails do Pacini. 

(2^83-1)(2^83+1)=2^166-1; por Fermat...; daí ele tentou verificar se 167
é fator do número pedido. 

Abraços 

Carlos victor 

Em 24/11/2015 20:13, Mauricio de Araujo escreveu: 

> Só para ser chato, o primo 167 caiu do céu? rsss (sem ofensas) 
> 
> No enunciado original não é mencionado o primo 167... 
> 
> Em 24 de novembro de 2015 16:48, Matheus Secco  
> escreveu:
> 
> Acredito que você possa usar resíduos quadráticos: 
> 
> (2 legendre p) = (-1)^(p^2-1)/8 
> 
> (2 legendre p) == 2^(p-1)/2 (mód p) 
> 
> Para p = 167, temos que (167^2-1)/8 é par. Logo (2 legendre 167) = 1. 
> Com isso, obtemos que 2^83 == 1 (mód 167). 
> 
> Abraços 
> 
> 2015-11-24 10:16 GMT-02:00 Pacini Bores :
> 
> Olá Marcone, 
> 
> Observe que 2^166-1 é divisível por 167; logo um dos fatores de 
> (2^83-1)(2^83+1) divide 167, já que 167 é primo. Só estou tentando provar que 
> é 2^83-1, que ainda não consegui. 
> 
> Pacini 
> 
> Em 24/11/2015 7:32, marcone augusto araújo borges escreveu: 
> Mostre que 2^83 - 1 não é primo 
> -- 
> Esta mensagem foi verificada pelo sistema de antivírus e 
> acredita-se estar livre de perigo. 
> -- 
> Esta mensagem foi verificada pelo sistema de antivírus e 
> acredita-se estar livre de perigo.

-- 
Esta mensagem foi verificada pelo sistema de antivírus e 
acredita-se estar livre de perigo. 

 -- 

Abraços 

oɾnɐɹɐ ǝp oıɔıɹnɐɯ 

-- 
Esta mensagem foi verificada pelo sistema de antivrus e 
acredita-se estar livre de perigo. 
-- 
Esta mensagem foi verificada pelo sistema de antiv�rus e
 acredita-se estar livre de perigo.



Re: [obm-l] Conicas

2015-10-31 Por tôpico Carlos Victor
 

Usando a ideia do Pacini, observe que dy/dx =0 para x=1 e y=0; dy/dx não
existe para x =-1/2 e que o eixo de simetria passa pelo ponto(-1/2,0) ;
ou seja o eixo de simetria é dado por y= -x-1/2. Fazendo a intersecção
dessa reta com a curva dada, encontramos x=-1/8 e y= -3/8, que são as
coordenadas do vértice. 

Abraços 

Carlos Victor 

Em 29/10/2015 23:01, Douglas Oliveira de Lima escreveu: 

> Olá caros amigos, gostaria de uma ajuda no seguinte problema: 
> 
> PROBLEMA: Encontrar a abscissa da parábola de equação 
> x^2+2xy+y^2-2x+4y+1=0. 
> 
> OBS: Essa questão caiu na prova do ITA acho que de 2012, e vi uma solução que 
> envolvia limites do qual não compreendi muito bem. 
> Sei portanto como usar a rotação de eixos e também através de diagonalização. 
> Mas gostaria de saber se existe outro modo de chegar a tal abscissa. 
> 
> Desde já obrigado. 
> Forte abraço do Douglas Oliveira. 
> -- 
> Esta mensagem foi verificada pelo sistema de antivrus e 
> acredita-se estar livre de perigo.
 
-- 
Esta mensagem foi verificada pelo sistema de antiv�rus e
 acredita-se estar livre de perigo.



Re: [obm-l] Limites

2015-09-08 Por tôpico Carlos Victor
Oi  Israel,
lim(n!/((n^n).(e^(-n)).(sqrt(2.pi.n))) = 1( relação de Stirling)  e use  o
fato de que lim (n^(1/n))=1.

Abraços

Carlos  Victor

Em 8 de setembro de 2015 21:03, Israel Meireles Chrisostomo <
israelmchrisost...@gmail.com> escreveu:

> Como posso provar de forma simples que ((2n)!/(n!)²)^{1/n}=4?Estou
> dependendo desse resultado para calcular um outro limite...
>
>
> --
> Esta mensagem foi verificada pelo sistema de antivírus e
> acredita-se estar livre de perigo.

-- 
Esta mensagem foi verificada pelo sistema de antiv�rus e
 acredita-se estar livre de perigo.



[obm-l] Re: [obm-l] indução

2015-06-28 Por tôpico Carlos Victor
Observar que o enunciado é 3^(1/3), ok ?

Em 28 de junho de 2015 12:03, Carlos Victor 
escreveu:

> Oi Marcone, irei resumir .
>
> Inicialmente a prova de que  n^3<3^n ou igual. Por indução:
>
> 3^(n+1) = 3.3^n > ou igual  que 3.n^3 = n^3+3n^2+3n + (n-3).n^2 +
> (n^2-3).n > n^3+3n^2+3n+1 = (n+1)^3.
>
> Suponha agora  que m
> PS:
> Esta questão foi da AMM, 1970,p 768, problem E2190, proposed by Harry
> Pollard, Purdue University , solved by Charles Wexler, Arizona State
> University, and 118 others.
>
> Abraços
>
> Carlos  Victor
>
> Em 28 de junho de 2015 11:31,  escreveu:
>
>> Qual a necessidade de escrever "n^1" ao invés de "n"? É algo da questão
>> mesmo?
>>
>> Enviado do meu iPhone
>>
>> Em 28/06/2015, às 11:17, marcone augusto araújo borges <
>> marconeborge...@hotmail.com> escreveu:
>>
>> Prove por indução que n^1/n < = 3^1/3, para n > = 2. Mostre que um dos
>> números
>> n^1/m ou m^1/n é maior que ou igual a 3, m e  naturais
>>
>> --
>> Esta mensagem foi verificada pelo sistema de antivírus e
>> acredita-se estar livre de perigo.
>>
>>
>> --
>> Esta mensagem foi verificada pelo sistema de antivírus e
>> acredita-se estar livre de perigo.
>>
>
>

-- 
Esta mensagem foi verificada pelo sistema de antiv�rus e
 acredita-se estar livre de perigo.



[obm-l] Re: [obm-l] indução

2015-06-28 Por tôpico Carlos Victor
Oi Marcone, irei resumir .

Inicialmente a prova de que  n^3<3^n ou igual. Por indução:

3^(n+1) = 3.3^n > ou igual  que 3.n^3 = n^3+3n^2+3n + (n-3).n^2 + (n^2-3).n
> n^3+3n^2+3n+1 = (n+1)^3.

Suponha agora  que m escreveu:

> Qual a necessidade de escrever "n^1" ao invés de "n"? É algo da questão
> mesmo?
>
> Enviado do meu iPhone
>
> Em 28/06/2015, às 11:17, marcone augusto araújo borges <
> marconeborge...@hotmail.com> escreveu:
>
> Prove por indução que n^1/n < = 3^1/3, para n > = 2. Mostre que um dos
> números
> n^1/m ou m^1/n é maior que ou igual a 3, m e  naturais
>
> --
> Esta mensagem foi verificada pelo sistema de antivírus e
> acredita-se estar livre de perigo.
>
>
> --
> Esta mensagem foi verificada pelo sistema de antivírus e
> acredita-se estar livre de perigo.
>

-- 
Esta mensagem foi verificada pelo sistema de antiv�rus e
 acredita-se estar livre de perigo.



[obm-l] Re: [obm-l] Área da Cicloide

2015-05-24 Por tôpico Carlos Victor
Oi  Eduardo, existe um texto no endereço a seguir. Verifique se é o que
você deseja.
http://www.apm.pt/apm/foco98/activ9.html

Abraços

Carlos  Victor

Em 24 de maio de 2015 18:46, Eduardo Henrique 
escreveu:

> Eu lendo um livro de história da matemática vi que Torricelli e Wren
> conseguiram demonstrar que a área sob um arco de cicloide é 3x a área do
> circulo que a gera utilizando o método da exaustão! Alguém saberia me
> indicar onde conseguir essas demonstrações ou até mesmo me dar uma luz em
> como faze-la?
>
> Att
>
> Eduardo
>
> --
> Esta mensagem foi verificada pelo sistema de antivírus e
> acredita-se estar livre de perigo.
>

-- 
Esta mensagem foi verificada pelo sistema de antiv�rus e
 acredita-se estar livre de perigo.



Re: [obm-l] Tabuleiro 3x3 com 4 cores

2015-03-30 Por tôpico Carlos Victor
Acredito que  ideia do Bob Roy é o mais rápida para obter a solução.

Carlos  Victor

Em 30 de março de 2015 10:39, Pacini Bores 
escreveu:

> Sim Pedro, esta é uma solução; ou seja, há possibilidade de se usar até
> quatro cores.
>
> Pacini
>
> Em 30 de março de 2015 10:23, Pedro José  escreveu:
>
>> Bom dia!
>>
>> Uma dúvida há necessidade de se usar as quatro cores ou há a
>> possibilidade de se usar até quatro cores?
>>
>> Por exemplo,
>>
>> 0 1 0
>> 1 0 1
>> 0 1 0
>>
>> onde 0 e 1 representam duas cores distintas, seria uma solução?
>>
>> Saudações,
>>
>> PJMS
>>
>>
>>
>>
>>
>> Em 29 de março de 2015 11:26, Bob Roy  escreveu:
>>
>>> Olá, O melhor para este problema é utlizar  o que o grande mestre
>>> Morgado falava : devemos inicialmente eliminar as dificuldades.
>>>
>>> Considerando uma matriz 3x3 , temos que os quadradinhos a12, a21, a23 e
>>> a32 não poderão ter todas as cores diferentes.
>>>
>>> Comece fazendo a análise com  duas cores iguais, três cores iguais e
>>> depois quatro cores iguais para essas posições.
>>>
>>> A análise ficará menos trabalhosa .
>>>
>>> Farei as contas e depois eu posto o resultado.
>>>
>>> Roy
>>>
>>>
>>> Em 28 de março de 2015 10:22, Carlos Victor 
>>> escreveu:
>>>
>>>> Comece pelo centro e pelas laterais, isto deve diminuir as
>>>> dificuldades. Abrirão vários casos para serem analisados.
>>>>
>>>> E se  não me engano, esta questão tem como origem  não considerando os
>>>> quadrados pelos vértices com as mesmas cores. Neste  caso a análise fica
>>>> mais silmplificada.
>>>>
>>>> Abraços
>>>>
>>>> Carlos Victor
>>>>
>>>> Em 28 de março de 2015 09:38, Pacini Bores 
>>>> escreveu:
>>>>
>>>>> Olá pessoal,  como pensar nesta ?
>>>>>
>>>>> De quantas maneiras podemos pintar um tabuleiro 3x3 com 4 cores de tal
>>>>> forma que não tenhamos cores adjacentes ?
>>>>>
>>>>> Nota : em diagonal não é considerado adjacente.
>>>>>
>>>>> Agradeço desde já
>>>>>
>>>>> Pacini.
>>>>>
>>>>> --
>>>>> Esta mensagem foi verificada pelo sistema de antivírus e
>>>>> acredita-se estar livre de perigo.
>>>>
>>>>
>>>>
>>>> --
>>>> Esta mensagem foi verificada pelo sistema de antivírus e
>>>> acredita-se estar livre de perigo.
>>>>
>>>
>>>
>>> --
>>> Esta mensagem foi verificada pelo sistema de antivírus e
>>> acredita-se estar livre de perigo.
>>>
>>
>>
>> --
>> Esta mensagem foi verificada pelo sistema de antivírus e
>> acredita-se estar livre de perigo.
>>
>
>
> --
> Esta mensagem foi verificada pelo sistema de antivírus e
> acredita-se estar livre de perigo.
>

-- 
Esta mensagem foi verificada pelo sistema de antiv�rus e
 acredita-se estar livre de perigo.



Re: [obm-l] Re:

2015-03-30 Por tôpico Carlos Victor
Olá Gabriel, esta é do livro do Gandhi :

(x^2+2)^2 = 4(x-2)^2 e daí .

Abraços

Carlos  Victor

Em 30 de março de 2015 07:16, Carlos Victor 
escreveu:

> Tente completar quadrados.
>
> Abraços
>
> Carlos Victor
>
> Em 29 de março de 2015 21:27, Gabriel Tostes 
> escreveu:
>
>> AlguÃĐm me ajuda a responder?
>> determine as raízes reais da equaçÃĢo:
>> X^4 + 16x - 12 = 0
>> --
>> Esta mensagem foi verificada pelo sistema de antivírus e
>>  acredita-se estar livre de perigo.
>>
>>
>> =
>> Instruįões para entrar na lista, sair da lista e usar a lista em
>> http://www.mat.puc-rio.br/~obmlistas/obm-l.html
>> =
>>
>
>
> --
> Esta mensagem foi verificada pelo sistema de antivírus e
> acredita-se estar livre de perigo.

-- 
Esta mensagem foi verificada pelo sistema de antiv�rus e
 acredita-se estar livre de perigo.



[obm-l] Re:

2015-03-30 Por tôpico Carlos Victor
Tente completar quadrados.

Abraços

Carlos Victor

Em 29 de março de 2015 21:27, Gabriel Tostes  escreveu:

> AlguÃĐm me ajuda a responder?
> determine as raízes reais da equaçÃĢo:
> X^4 + 16x - 12 = 0
> --
> Esta mensagem foi verificada pelo sistema de antivírus e
>  acredita-se estar livre de perigo.
>
>
> =
> Instruįões para entrar na lista, sair da lista e usar a lista em
> http://www.mat.puc-rio.br/~obmlistas/obm-l.html
> =
>

-- 
Esta mensagem foi verificada pelo sistema de antiv�rus e
 acredita-se estar livre de perigo.



Re: [obm-l] Tabuleiro 3x3 com 4 cores

2015-03-28 Por tôpico Carlos Victor
Comece pelo centro e pelas laterais, isto deve diminuir as dificuldades.
Abrirão vários casos para serem analisados.

E se  não me engano, esta questão tem como origem  não considerando os
quadrados pelos vértices com as mesmas cores. Neste  caso a análise fica
mais silmplificada.

Abraços

Carlos Victor

Em 28 de março de 2015 09:38, Pacini Bores 
escreveu:

> Olá pessoal,  como pensar nesta ?
>
> De quantas maneiras podemos pintar um tabuleiro 3x3 com 4 cores de tal
> forma que não tenhamos cores adjacentes ?
>
> Nota : em diagonal não é considerado adjacente.
>
> Agradeço desde já
>
> Pacini.
>
> --
> Esta mensagem foi verificada pelo sistema de antivírus e
> acredita-se estar livre de perigo.

-- 
Esta mensagem foi verificada pelo sistema de antiv�rus e
 acredita-se estar livre de perigo.



Re: [obm-l] Problema IMO

2015-03-10 Por tôpico Carlos Victor
Oi Israel, no link

http://www.artofproblemsolving.com/wiki/index.php/1985_IMO_Problems/Problem_4,
 vc encontra a solução, ok ?


Abraços


Carlos  Victor



Em 10 de março de 2015 21:46, Israel Meireles Chrisostomo <
israelmchrisost...@gmail.com> escreveu:

> Alguém poderia me ajudar nessa questão envolvendo o princípio da casa dos
> pombos?
> "Dado um conjunto M com 1985 inteiros positivos distintos, nenhum dos
> quais tem divisores maiores do que 23, mostre que há 4 elementos em M cujo
> produto é uma quarta potência."
> Pensei em usar que de 2 a 23 tem 9 números primos, mas não sei bem como
> aplicar o princípio, se alguém pudesse me ajudar serei grato.
>
> --
> Esta mensagem foi verificada pelo sistema de antivírus e
> acredita-se estar livre de perigo.

-- 
Esta mensagem foi verificada pelo sistema de antiv�rus e
 acredita-se estar livre de perigo.



[obm-l] Re: [obm-l] Re: [obm-l] OBM NÍVEL 3 TERCEIRA FASE PRIMEIRO DIA

2014-11-17 Por tôpico Carlos Victor
Oi grande Douglas !!

Como sempre postando bons problemas para a nossa comunidade.

Vamos lá :

Sejam M,N,R e Q os incentros dos triângulos ABP, BPC, CPD e APD
respectivamente.

Sejam S, T, U e V  os incentros dos triângulos ABC,BCD, ACD e ABD
respectivamente.

1º) mostre que  MNRQ é um losango. Mostre também que os raios dos círculos
inscritos em ABC e ADC são iguais; da mesma forma dos triângulos ABD e BCD.

2º) depois mostre que  AM/MS =  AQ/QU  e que  SN/NC = RU/RT .

3º) como consequência SU é paralelo a BD e que VT é paralelo  a AC.

4º) mostre que mostre que MS/AM = UR/RC.

5º) mostre que o ângulo MSN = ângulo QUR ; da mesma forma  ângulo NTR  =
ângulo MVQ.

6º) conclua então que ASCU é um paralelogramo.

7º) conclua daí que pelo fato de PN = PQ e  MP = PR , teremos S pertencente
a BD e  V pertencente a AC.

8º) Como BP é bissetriz e intersecta AC no ponto médio, temos que AB=AC e
BP é perpendicular a BC.

9º) da mesma forma ACD é isósceles.

10º) ídem para BCD e ABD .


Conclusão : ABCD é um losango.. UFA .


Abraços

Carlos  Victor


Em 30 de outubro de 2014 12:22, Esdras Muniz 
escreveu:

> Opa, eu tinha entendido círculos circunscritos... Foi mal.
>
> Em 30 de outubro de 2014 11:02, Esdras Muniz 
> escreveu:
>
>>
>>
>> Em 29 de outubro de 2014 22:50, Douglas Oliveira de Lima <
>> profdouglaso.del...@gmail.com> escreveu:
>>
>>
>>>  *PROBLEMA 1 *
>>>
>>> Seja *ABCD *um quadrilátero convexo e seja *P *a interseção das
>>> diagonais *AC *e *BD*. Os raios dos círculos inscritos nos triângulos
>>> *ABP*, *BCP*, *CDP *e *DAP *são iguais. Prove que *ABCD *é um losango.
>>>
>>>
>>> Como poderíamos fazer esse problema?
>>>
>>>
>>>
>>>
>>> --
>>> Esta mensagem foi verificada pelo sistema de antivírus e
>>> acredita-se estar livre de perigo.
>>
>>
>>
>>
>> --
>> Esdras Muniz Mota
>> Graduando em Matemática Bacharelado
>> Universidade Federal do Ceará
>>
>>
>>
>
>
> --
> Esdras Muniz Mota
> Graduando em Matemática Bacharelado
> Universidade Federal do Ceará
>
>
>
> --
> Esta mensagem foi verificada pelo sistema de antivírus e
> acredita-se estar livre de perigo.
>

-- 
Esta mensagem foi verificada pelo sistema de antiv�rus e
 acredita-se estar livre de perigo.



Re: [obm-l] Problema de Geometria

2014-11-03 Por tôpico Carlos Victor
Oi Mariana,

Seja   x o  ângulo DCA . Aplicando a lei dos senos nos triângulos ACD e BCD
, vc encontrará a seguinte relação :

senx = 2sen(x+20).cos80.

Transformando em soma teremos : senx = sen(x+100) + sen(x-60).

Jogando para a esquerda o sen(x-60), teremos  senx - sen(x-60) =
sen(x+100); ou  seja :

2sen(30)cos(x-30) = sen(x+100) ; ou seja ;  sen(120-x) = sen(x+100) ; ou
seja :

x = 10º. Confira as contas, ok ?

Abraços

Carlos  Victor


PS : este problema se torna mais interessante, colocando o seguinte
enunciado :

No triângulo ABC, AB = AC . Um ponto D está sobre o lado AB e
AD = BC,  tal que  CD passe pelo circuncentro de ABC. Calcule os ângulos do
 triângulo.

Em 3 de novembro de 2014 20:21, Mariana Groff <
bigolingroff.mari...@gmail.com> escreveu:

> Boa Noite,
> Alguém poderia me ajudar no problema a seguir?
>
> No triângulo ABC, AB = AC e BÂC = 20º. Um ponto D está sobre o lado AB e
> AD = BC. Calcule o ângulo 
> Obrigada,
> Mariana
>
> --
> Esta mensagem foi verificada pelo sistema de antivírus e
> acredita-se estar livre de perigo.

-- 
Esta mensagem foi verificada pelo sistema de antiv�rus e
 acredita-se estar livre de perigo.



Re: [obm-l] Geometria.

2014-11-03 Por tôpico Carlos Victor
Oi  Pedro, esse  é um problema bem difícil e a solução, o Gandhi ( Antonio
Luis) me mostrou um tempo atrás ( 1997 se não me engano...). Vou tentar
escrevê-lo. Faça uma figura e acompanhe, ok ?

Vamos lá :

Vamos escolher dois pontos  M e N sobre BC, tais que N seja o simétrico de
E( ângulo em E igual a 18) em relação à bissetriz CF  e M o simétrico  de F
em relação à bissetriz  BE. Trace NI, NF  e trace ME.  Seja Q o encontro de
ME com FC. Conclua que os ângulos MQF, MQN e NQC são iguais a 60º. Como FN
é bissetriz do ângulo MFQ e NQ é bissetriz externa do ângulo MQF, temos
 que MN é bissetriz externa do ângulo QMF e daí encontre o ângulo interno
em B igual a 72º. Como o ângulo em A é 96º , temos que o ângulo interno em
C é igual 12º.

Donde  B- C = 60º, UFA !!!.

Caso não entenda alguma parte , escreva, ok ?


Abraços

Carlos  Victor

Em 3 de novembro de 2014 13:37, Pedro José  escreveu:

> Seja ABC um triângulo e E e F os pés das bissetrizes internas dos ângulos
> B e C respectivamente. Sabendo-se que os ângulos E e F do triângulo EIF,
> onde I é o incentro de ABC, medem 18 e 24 graus, calcule B-C.
>
> Alguém tem alguma ideia?
>
> Grato,
> PJMS
>
> --
> Esta mensagem foi verificada pelo sistema de antivírus e
> acredita-se estar livre de perigo.

-- 
Esta mensagem foi verificada pelo sistema de antiv�rus e
 acredita-se estar livre de perigo.



Re: [obm-l] Geometria

2014-10-25 Por tôpico Carlos Victor
Oi  Douglas,

Pense assim :

1) Mostre inicialmente que aplicando a lei dos senos para o triângulo OHA,
encontramos

cosB =2cosA.cosC., sabendo que AH = 2. OS, onde  S é o ponto médio de CB.

2) Sabendo que os lados do triângulo órtico são dados por :

Rsen2A, Rsen2B e Rsen2C e fazendo a semi-soma do primeiro com o último, e
utilizando (1),

conclua que esse triângulos tem as medidas dos lados em PA, ok ?

Nota : R é o raio do círculo circunscrito ao triângulo. ( confira as contas)


Abraços

Carlos Victor




Em 24 de outubro de 2014 07:07, Douglas Oliveira de Lima <
profdouglaso.del...@gmail.com> escreveu:

> Bom dia a todos, não vi solução para essa questão,
>
> Sejam H, O o ortocentro e o circuncentro do triÂngulo ABC. AD, BE e CF são
> as alturas relativas aos vértices A, B e C. Suponha que OH seja paralelo
> a AC. Mostre que os lados do triângulo DEF estão em progressão aritmética.
>
>
> Agradeço a ajuda!!
>
> Douglas Oliveira.
>
>
>
>
> --
> Esta mensagem foi verificada pelo sistema de antivírus e
> acredita-se estar livre de perigo.

-- 
Esta mensagem foi verificada pelo sistema de antiv�rus e
 acredita-se estar livre de perigo.



Re: [obm-l] Inteiros

2014-09-13 Por tôpico Carlos Victor
Oi  Marcone, essa é do Mathematical Morsels.

Já que 3abc é positivo, devemos ter  a^3 maior que b^3  e c^3.

Logo b escreveu:

> Determine todos os naturais a,b e c tais que a^3 - b^3 - c^3 = 3abc e a^2
> = 2(b+c)
>
> --
> Esta mensagem foi verificada pelo sistema de antivírus e
> acredita-se estar livre de perigo.
>

-- 
Esta mensagem foi verificada pelo sistema de antiv�rus e
 acredita-se estar livre de perigo.



Re: [obm-l] Problema de geometria!

2014-07-06 Por tôpico Carlos Victor
OI Douglas ,

Pensando neste problema, se usar a lei dos cossenos nos triângulos FCE, AFD
e DBE e usando o fato de que cos(90+B)= -senB ( não é muito trabalhoso);
 deixando na forma de quadrados não é difícil de concluir que 4 FE^2= ED^2
e que 4DF^2 = 3ED^2 ; ou seja o triângulo EFD é retângulo e que que os
ângulos pedidos são 90º e 30º , ok ?

Pelo visto vc está querendo uma solução com algum traçado mágico, não é
verdade?
Estarei pensando, ok  Douglas! Estou desconfiado que deve ter alguma coisa
com o Teorema de Napoleão ...

Vamos tentar, pois deve ser assaz interessante tal traçado.

Abraços

Carlos Victor


Em 13 de junho de 2014 16:09, Douglas Oliveira de Lima <
profdouglaso.del...@gmail.com> escreveu:

> Olá caros amigos, me encontro mais uma vez com um pequeno problema de
> geometria no qual estou com uma solução muito absurda(muito trabalho
> braçal), gostaria de uma ajuda com outras soluções, desde já agradeço a
> colaboração dos senhores.
>
> PROBLEMA:
> Considere um triângulo ABC, são construídos externamente os triângulos ADB
> e BCE de forma que ADB=BEC=90 GRAUS E DAB=EBC=30 graus. No segmento AC
> marca-se o ponto F tal que AF=3FC. Calcular os ângulos DFE e FDE.
>
> Douglas Oliveira de Lima
>
> --
> Esta mensagem foi verificada pelo sistema de antivírus e
> acredita-se estar livre de perigo.

-- 
Esta mensagem foi verificada pelo sistema de antiv�rus e
 acredita-se estar livre de perigo.



Re: [obm-l] Provar perpendicularidade em Geometria Plana

2014-05-25 Por tôpico Carlos Victor
Oi Hermann, certamente a sua solução é bonita e mais simples.

Abraços

Carlos Victor


Em 25 de maio de 2014 16:02, Hermann  escreveu:

> A solução do Carlos é excelente.
>
> A minha solução é só com arco capaz
>
> Como o ângulo BEC e BFC =90 temos a circunferência BEFC e nela observamos
> pelo arco FC que os ângulos B e E são iguais.
>
> Agora (como já visto pelo Carlos) na circunferência BAC o ângulo A = 90 - B
>
>
> E o  segmento OA é perpendicular ao segmento EF.
>
> Abraços
> Hermann
>
>
>
> - Original Message - From: Martins Rama
> To: OBM-L
> Sent: Sunday, May 25, 2014 11:03 AM
> Subject: [obm-l] Provar perpendicularidade em Geometria Plana
>
>
>
> Caros amigos, alguém me auxilia nessa demonstração de Geom Plana? É do
> livro da SBM do Antonio Caminha Muniz Neto.
> Abraço a todos.
> Martins Rama.
>
> Seja ABC um triângulo acutângulo de circuncentro O. Se E e F são os pés
> das alturas relativas aos vértices B e C, respectivamente, prove que o
> segmento OA é perpendicular ao segmento EF.
> --
> Esta mensagem foi verificada pelo sistema de antiv�rus e
>
> acredita-se estar livre de perigo.
>
> --
> Esta mensagem foi verificada pelo sistema de antivírus e
> acredita-se estar livre de perigo.
>
> =
> Instru�ões para entrar na lista, sair da lista e usar a lista em
> http://www.mat.puc-rio.br/~obmlistas/obm-l.html
> =
>

-- 
Esta mensagem foi verificada pelo sistema de antiv�rus e
 acredita-se estar livre de perigo.



Re: [obm-l] Provar perpendicularidade em Geometria Plana

2014-05-25 Por tôpico Carlos Victor
Oi Martins, esqueci de dizer que o ponto  R é a interseccão de OA e EF, ok ?

Abraços
Carlos Victor


Em 25 de maio de 2014 13:31, Carlos Victor escreveu:

> Oi Martins, Observe o seguinte :
>
> Os segmentos AE e AF são respectivamente : c.cosA e bcosA.
> Observe agora que os triângulos ABC e AEF são semelhantes, por possuirem
>  os lados AC e AB  com razões iguais aos lados AE e AF e, claro um ângulo
> em comum.
>
> Donde o ângulo FEA = ângulo em B.
>
>  Como o ângulo OAC = 90 - B, teremos o
> ângulo ERA = 90 graus, ok ?
>
> Abraços
>
> Victor
>
>
> Em 25 de maio de 2014 11:03, Martins Rama  escreveu:
>
> Caros amigos, alguém me auxilia nessa demonstração de Geom Plana? É do
>> livro da SBM do Antonio Caminha Muniz Neto.
>> Abraço a todos.
>> Martins Rama.
>>
>> Seja ABC um triângulo acutângulo de circuncentro O. Se E e F são os pés
>> das alturas relativas aos vértices B e C, respectivamente, prove que o
>> segmento OA é perpendicular ao segmento EF.
>> --
>> Esta mensagem foi verificada pelo sistema de antivírus e
>> acredita-se estar livre de perigo.
>>
>
>

-- 
Esta mensagem foi verificada pelo sistema de antiv�rus e
 acredita-se estar livre de perigo.



Re: [obm-l] Provar perpendicularidade em Geometria Plana

2014-05-25 Por tôpico Carlos Victor
Oi Martins, Observe o seguinte :

Os segmentos AE e AF são respectivamente : c.cosA e bcosA.
Observe agora que os triângulos ABC e AEF são semelhantes, por possuirem
 os lados AC e AB  com razões iguais aos lados AE e AF e, claro um ângulo
em comum.

Donde o ângulo FEA = ângulo em B.

 Como o ângulo OAC = 90 - B, teremos o
ângulo ERA = 90 graus, ok ?

Abraços

Victor


Em 25 de maio de 2014 11:03, Martins Rama  escreveu:

> Caros amigos, alguém me auxilia nessa demonstração de Geom Plana? É do
> livro da SBM do Antonio Caminha Muniz Neto.
> Abraço a todos.
> Martins Rama.
>
> Seja ABC um triângulo acutângulo de circuncentro O. Se E e F são os pés
> das alturas relativas aos vértices B e C, respectivamente, prove que o
> segmento OA é perpendicular ao segmento EF.
> --
> Esta mensagem foi verificada pelo sistema de antivírus e
> acredita-se estar livre de perigo.
>

-- 
Esta mensagem foi verificada pelo sistema de antiv�rus e
 acredita-se estar livre de perigo.



Re: [obm-l] Re: Mais uma que quero compartilhar!!

2014-05-17 Por tôpico Carlos Victor
Esse é um dos problemas mais lindo que o  meu grande companheiro Gandhi me
apresentou.
Abraços Douglas.

Carlos Victor


Em 15 de maio de 2014 23:16, Douglas Oliveira de Lima <
profdouglaso.del...@gmail.com> escreveu:

> Eu nao sei se deu pra compreender direito a expressão , mas acho que
> escrevi certinho, o resultado da 3.
>
>
>
> Em 15 de maio de 2014 17:45, Douglas Oliveira de Lima <
> profdouglaso.del...@gmail.com> escreveu:
>
> Essa vai em homenagem a meu grande amigo Gandhi(Antonio Luiz Santos) que
>> me ensinou como fazer, quero dizer também que essa lista da obm(do qual
>> usamos para discutir questões de olimpíadas e outras questões
>> interessantes) esta sendo pra mim muito gratificante neste momento, porque
>> nos que gostamos de matemática, gostamos de resolver questões ate em papel
>> de guardanapo no restaurante,pois desde que me mudei para Brasilia não
>> encontrei professores aqui como os que tive a oportunidade de  conhecer no
>> Rio de Janeiro como Gandhi, Carlos Victor, Eduardo Mauro, Eduardo Wagner,
>> Haroldo, Poncio, Ivan, Bandeira, e claro não poderia esquecer do grande
>> Alvaro, e que me ajudaram a crescer dentro desta área. Existem outros que
>> conheci, mas hoje o mérito vai pra eles,professores humildes, que nunca me
>> negaram sequer uma questão, ajudando o amigo a crescer para que um dia
>> possamos derrubar essa grande massa de professores ruins(que não gostam de
>> estudar) do mercado,e quem sabe assim incentivando a futura geração a
>> curtir matemática.
>>
>> Encontrar o valor de (1+2(1+3(1+4(1+...)^(1/2))^(1/2))^(1/2))^(1/2)
>>
>
>
> --
> Esta mensagem foi verificada pelo sistema de antivírus e
> acredita-se estar livre de perigo.
>

-- 
Esta mensagem foi verificada pelo sistema de antiv�rus e
 acredita-se estar livre de perigo.



[obm-l] Re: [obm-l] Retângulo de incentros num quadrilatero inscritível

2014-04-05 Por tôpico Carlos Victor
No email anterior, onde estiver "a" , lê-se I(a)...; ok ?

Abraços

Carlos Victor


Em 5 de abril de 2014 14:27, Carlos Victor escreveu:

> Olá Martins,
>
> Vamos mostrar inicialmente que o ângulo (abc) é reto, então para os outros
>  a demonstração é idêntica.
>
> 1) Como ABCD é inscritível, então os ângulos ADB e ACB são iguais. Da
> mesma forma  que os ângulos ABD e DCA são iguais, ok ?
>
> 2) Vamos denominar o seguinte :
>
> DCA = ABD = 2x ; CAD= DBC=2z ; ADB = ACB = 2y ; BDC= 2t .
>
> 3) Observe que o ângulo DbC = 90+z = DaC ; donde se conclui que o
> quadrilátero abDC é inscritível, ok ?
> Logo os ângulos Cba e  CDa são iguais, ou seja, Cba= t.
>
> 4) Pelo mesmo motivo, os ângulos  Dcb = DAb= z   e ADc = Abc = y.
>
> 5) Agora, olhando para o vértice "b"  e tomando  a soma dos ângulos  em
> torno dele  igual a 360 graus e  fazendo x+y+z +t =90 graus, encontraremos
> o ângulo b = 90 graus , do quadrilátero abcd.
>
> Para os outros vértices , utilizamos a mesma ideia.
>
> Abraços
>
> Carlos  Victor
>
>
> Em 4 de abril de 2014 15:59,  escreveu:
>
> Olá amigos da lista. Não estou conseguindo resolver esta de Geometria
>> Plana:
>>
>> Seja ABCD um quadrilátero inscritível e sejam I(a), I(b), I(c) e I(d),
>> respectivamente, os incentros dos triângulos BCD, CDA, DAB e ABC. Prove que
>> o quadrilátero I(a)I(b)I(c)I(d) é um retângulo.
>>
>> Alguém pode ajudar?
>>
>> Martins Rama.
>>
>>
>> --
>> Esta mensagem foi verificada pelo sistema de antivírus e
>> acredita-se estar livre de perigo.
>>
>
>

-- 
Esta mensagem foi verificada pelo sistema de antivírus e
 acredita-se estar livre de perigo.



[obm-l] Re: [obm-l] Retângulo de incentros num quadrilatero inscritível

2014-04-05 Por tôpico Carlos Victor
Olá Martins,

Vamos mostrar inicialmente que o ângulo (abc) é reto, então para os outros
 a demonstração é idêntica.

1) Como ABCD é inscritível, então os ângulos ADB e ACB são iguais. Da mesma
forma  que os ângulos ABD e DCA são iguais, ok ?

2) Vamos denominar o seguinte :

DCA = ABD = 2x ; CAD= DBC=2z ; ADB = ACB = 2y ; BDC= 2t .

3) Observe que o ângulo DbC = 90+z = DaC ; donde se conclui que o
quadrilátero abDC é inscritível, ok ?
Logo os ângulos Cba e  CDa são iguais, ou seja, Cba= t.

4) Pelo mesmo motivo, os ângulos  Dcb = DAb= z   e ADc = Abc = y.

5) Agora, olhando para o vértice "b"  e tomando  a soma dos ângulos  em
torno dele  igual a 360 graus e  fazendo x+y+z +t =90 graus, encontraremos
o ângulo b = 90 graus , do quadrilátero abcd.

Para os outros vértices , utilizamos a mesma ideia.

Abraços

Carlos  Victor


Em 4 de abril de 2014 15:59,  escreveu:

> Olá amigos da lista. Não estou conseguindo resolver esta de Geometria
> Plana:
>
> Seja ABCD um quadrilátero inscritível e sejam I(a), I(b), I(c) e I(d),
> respectivamente, os incentros dos triângulos BCD, CDA, DAB e ABC. Prove que
> o quadrilátero I(a)I(b)I(c)I(d) é um retângulo.
>
> Alguém pode ajudar?
>
> Martins Rama.
>
>
> --
> Esta mensagem foi verificada pelo sistema de antivírus e
> acredita-se estar livre de perigo.
>

-- 
Esta mensagem foi verificada pelo sistema de antivírus e
 acredita-se estar livre de perigo.



[obm-l] Re: [obm-l] Re: [obm-l] Re: [obm-l] Re: [obm-l] Re: [obm-l] Re: [obm-l] Re: [obm-l] Re: [obm-l] RE: [obm-l] Re: [obm-l] Três de inteiros

2014-02-24 Por tôpico Carlos Victor
Pacini,

 vc tem que  retirar os casos de que  x+y+z =0 , ok ?

Carlos Victor


Em 24 de fevereiro de 2014 16:44, Pacini Bores escreveu:

> Olá pessoal, posso fazer o que está descrito a seguir  no terceiro
> problema ?
>
> Sabemos que x^2+y^2+z^2 *>* xy+xz+yz  e na hipótese de que  xy+xz+yz não
> seja nulo, teremos :
>
> (x^2 + y^2 + z^2)/2(xy+yz+xz) *>* 1/2 , para xy+xz+yz  > 0   e
>
> (x^2 + y^2 + z^2)/2(xy+yz+xz) *>* -1/2 , para xy+xz+yz  < 0 .
>
> Daí  F(x,y,z)  varia de  [-1/2, 0[  união  [1/2,+infinito[ .
>
> Pacini
>
>
>
>
>
> Em 24 de fevereiro de 2014 12:43, terence thirteen <
> peterdirich...@gmail.com> escreveu:
>
> Quanto ao último,
>>
>> 3) Se x,y,z são números reais não nulos,com x+y+z também não nulo
>> Calcule os valores possíveis da expressão F(x,y,z) = (x^2 + y^2 +
>> z^2)/2(xy+yz+xz)
>>
>> Acho que dá para aplicar rearranjo, não?
>>
>> Primeiro, por homogeneidade, supunhetemos que x+y+z=1. Segundo, por
>> simetria, x>=y>=z.
>>
>> Temos x^2+y^2+z^2 >= x^2+yz+zy, afinal basta subtrair:
>>
>> (x^2-x^2) + (y^2-yz) + (z^2-zy) = 0 + y(y-z) + z(z-y) = (z-y)^2 >=0
>>
>> E também,
>>
>> x^2+yz+zy >= xy+yz+zy
>>
>> Demonstre da mesma forma!
>>
>> Agora, temos que ver os sinais...
>>
>> Em 21/02/14, Tarsis Esau escreveu:
>> > Bernado, vc tinha razão. resolvendo 2) a resposta é (-33, -33).
>> >
>> > Desenvolvendo 2) (m + n)² + (m + n).33 + 33² = 3mn, vamos chegar a
>> >
>> > m² -mn + 33m + n² + 33n + 33² = 0
>> >
>> > Resolvendo em função de n, teremos um delta [(n-33)² - 4.(n² + 33n +
>> 33²)]
>> > = -3n² -6.33n - 3.33²,
>> >
>> > Sendo que -3n² -6.33n - 3.33² >=0
>> >
>> > Estudando o sinal da parábola, temos que a concavidade deve estar
>> voltada
>> > para baixo, assumindo assim um máximo
>> >
>> > O delta desta nova equação é 0 e ela apresenta máximo em n = -33.
>> > Substituindo-se em 2), m = -33.
>> >
>> >
>> >
>> >
>> >
>> >
>> >
>> > 2014-02-21 17:02 GMT-03:00 Bernardo Freitas Paulo da Costa <
>> > bernardo...@gmail.com>:
>> >
>> >> 2014-02-21 14:24 GMT-03:00 Tarsis Esau :
>> >> > Fiz a segunda, vou tentar fazer a terceira :)
>> >> >
>> >> > m³ + n³ + 99mn = 33³
>> >> >
>> >> > (m + n)³ - 3m²n - 3mn² + 99mn = 33³
>> >> > (m + n)³ - 33³ = 3mn.[(m + n) - 33]
>> >> > [(m +n) - 33].[(m + n)² + (m +n).33 + 33²] = 3mn.[(m+n) - 33]
>> >> >
>> >> > Assim, temos
>> >> >
>> >> > 1) m + n - 33 = 0
>> >> >
>> >> > e
>> >>
>> >> Deveria ser "ou", mas você agiu como se fosse "ou". Mas isso é menos
>> >> importante que o meu próximo comentário.
>> >>
>> >> > 2) (m + n)² + (m + n).33 + 33² = 3mn
>> >> >
>> >> > De 1) temos todos os pares (x,y): (0,33); (1,32), ..., (32, 1), (33,
>> >> > 0).
>> >> > Todos os inteiros estão neste intervalo.
>> >> >
>> >> > Uma vez que , caso um m seja maior que 33, o n necessariamente deve
>> ser
>> >> > menor que zero, o que vai contra o enunciado de m.n >=0.
>> >> >
>> >> > Desse modo, não há necessidade de resolver 2).
>> >>
>> >> Claro que há. Pode ser que a equação 2 tenha uma solução com m = 20 e
>> >> n = 10 (sei lá) cuja soma não é 33. Se você tivesse obtido TODOS os
>> >> pares (a,b) com 0<=a<=33, 0 <=b<=33 como solução, aí tava certo. Mas
>> >> veja que essa é uma equação cúbica, portanto para cada "m" existem
>> >> três soluções "n" possíveis. É bastante provável que, se m é inteiro,
>> >> não haja muitas soluções com n inteiro, mas você tem que demonstrar
>> >> isso. Além disso, o enunciado diz que m.n >= 0, ou seja, pode ser que
>> >> m e n sejam NEGATIVOS! (mas talvez o enunciado tenha sido copiado
>> >> errado, e era para ser m E n >= 0).
>> >> --
>> >> Bernardo Freitas Paulo da Costa
>> >>
>> >> --
>> >> Esta mensagem foi verificada pelo sistema de antivírus e
>> >>  acredita-se estar livre de perigo.
>> >>
>> >>
>> >>
>> =
>> >> Instruções para entrar na lista, sair da lista e usar a lista em
>> >> http://www.mat.puc-rio.br/~obmlistas/obm-l.html
>> >>
>> =
>> >>
>> >
>> > --
>> > Esta mensagem foi verificada pelo sistema de antivírus e
>> >  acredita-se estar livre de perigo.
>> >
>> >
>>
>>
>> --
>> /**/
>> 神が祝福
>>
>> Torres
>>
>> --
>> Esta mensagem foi verificada pelo sistema de antivírus e
>>  acredita-se estar livre de perigo.
>>
>>
>> =
>> Instru�ões para entrar na lista, sair da lista e usar a lista em
>> http://www.mat.puc-rio.br/~obmlistas/obm-l.html
>> =
>>
>
>
> --
> Esta mensagem foi verificada pelo sistema de antivírus e
> acredita-se estar livre de perigo.
>

-- 
Esta mensagem foi verificada pelo sistema de antiv�rus e
 acredita-se estar livre de perigo.



Re: [obm-l] R^2=(BC^2+AH^2)/4

2014-02-18 Por tôpico Carlos Victor
Olá luís,

O enunciado é este com o seguinte detalhe : H é o ortocentro de ABC, daí AH
não é a altura, ok ?

Abraços

Victor


Em 17 de fevereiro de 2014 23:29, luiz silva
escreveu:

> É porque eu não vi o enunciado. Seria assim : Em qualquer triângulo ABC, a
> soma do quadrado do lado BC e do quadrado da altura AH, relativa a BC, é
> igual ao quadrado do diâmetro do círculo circunscrito?
>
> Abs
> Felipe
>
>
>   Em Segunda-feira, 17 de Fevereiro de 2014 19:02, Carlos Victor <
> victorcar...@globo.com> escreveu:
>  Sim Luís,
> Você pode encontrar essa relação em vários livros de geometria que fale
> sobre a reta de Euler,   que passa  pelo circuncentro, ortocentro e
> baricentro, ok ?
> Abraços
>
> Carlos Victor
>
>
> Em 17 de fevereiro de 2014 18:33, luiz silva 
> escreveu:
>
> Essa relação é valida em um triangulo qualquer ?
>
> Abs
> Felipe
>
>
>
>   Em Segunda-feira, 17 de Fevereiro de 2014 15:49, Carlos Victor <
> victorcar...@globo.com> escreveu:
>   Oi Luís,
> Apesar do enunciado não falar, H é o ortocentro do triângulo, ok ?
>
> Abraços
>
> Carlos Victor
>
>
> Em 16 de fevereiro de 2014 22:33, luiz silva 
> escreveu:
>
> AH é a altura relativa à BC?
>
>
>   Em Sábado, 15 de Fevereiro de 2014 17:30, Carlos Victor <
> victorcar...@globo.com> escreveu:
>  Oi Luís,
> digitei errado.
>
> Onde está AM lê-se  AH, ok ?
>
> Desculpe  o engano...
>
> Carlos  Victor
>
>
> Em 15 de fevereiro de 2014 16:53, Carlos Victor 
> escreveu:
>
> Oi Luís,
>
> Seja M o ponto médio de BC  e "O" o circuncentro do triângulo ABC. Prove
> inicialmente que AM= 2.OM <http://2.om/> e aplique Pitágoras no triângulo
> OMC, por exemplo.
> Daí sai legal a relação que tu queres, ok ?
>
> Para provar que AM = 2.OM <http://2.om/> , pense no alinhamento que
> existe entre o circuncentro, ortocentro e baricentro... .
>
> Abraços
>
> Carlos  Victor
>
>
> Em 13 de fevereiro de 2014 13:13, Luís  escreveu:
>
>  Sauda,c~oes,
>
> Como provar a relação abaixo?
>
> R^2=(BC^2+AH^2)/4
>
> Imaginei colocar os pontos B,C,H com as seguintes coordenadas:
>
> B=(0,0)  C=(a,0)  H=(h,y_H) A=(h,y_A)
>
> Daí a gente obtém o ponto H_c=(x,y) com régua e compasso e
> em seguida o ponto A. O circuncentro (O) é calculado e finalmente R.
>
> As contas não são legais com papel e lápis. Alguém poderia dar as
> coordenadas dos pontos A e (O) usando um programa de
> cálculo simbólico ?
>
> Obrigado.
>
> Luís
>
>
> --
> Esta mensagem foi verificada pelo sistema de antivírus e
> acredita-se estar livre de perigo.
>
>
>
>
> --
> Esta mensagem foi verificada pelo sistema de antivírus e
> acredita-se estar livre de perigo.
>
>
>
> --
> Esta mensagem foi verificada pelo sistema de antivírus e
> acredita-se estar livre de perigo.
>
>
>
> --
> Esta mensagem foi verificada pelo sistema de antivírus e
> acredita-se estar livre de perigo.
>
>
>
> --
> Esta mensagem foi verificada pelo sistema de antivírus e
> acredita-se estar livre de perigo.
>
>
>
> --
> Esta mensagem foi verificada pelo sistema de antivírus e
> acredita-se estar livre de perigo.
>
>
>
> --
> Esta mensagem foi verificada pelo sistema de antivírus e
> acredita-se estar livre de perigo.
>

-- 
Esta mensagem foi verificada pelo sistema de antivírus e
 acredita-se estar livre de perigo.



Re: [obm-l] R^2=(BC^2+AH^2)/4

2014-02-17 Por tôpico Carlos Victor
Sim Luís,
Você pode encontrar essa relação em vários livros de geometria que fale
sobre a reta de Euler,   que passa  pelo circuncentro, ortocentro e
baricentro, ok ?
Abraços

Carlos Victor


Em 17 de fevereiro de 2014 18:33, luiz silva
escreveu:

> Essa relação é valida em um triangulo qualquer ?
>
> Abs
> Felipe
>
>
>
>   Em Segunda-feira, 17 de Fevereiro de 2014 15:49, Carlos Victor <
> victorcar...@globo.com> escreveu:
>  Oi Luís,
> Apesar do enunciado não falar, H é o ortocentro do triângulo, ok ?
>
> Abraços
>
> Carlos Victor
>
>
> Em 16 de fevereiro de 2014 22:33, luiz silva 
> escreveu:
>
> AH é a altura relativa à BC?
>
>
>   Em Sábado, 15 de Fevereiro de 2014 17:30, Carlos Victor <
> victorcar...@globo.com> escreveu:
>  Oi Luís,
> digitei errado.
>
> Onde está AM lê-se  AH, ok ?
>
> Desculpe  o engano...
>
> Carlos  Victor
>
>
> Em 15 de fevereiro de 2014 16:53, Carlos Victor 
> escreveu:
>
> Oi Luís,
>
> Seja M o ponto médio de BC  e "O" o circuncentro do triângulo ABC. Prove
> inicialmente que AM= 2.OM <http://2.om/> e aplique Pitágoras no triângulo
> OMC, por exemplo.
> Daí sai legal a relação que tu queres, ok ?
>
> Para provar que AM = 2.OM <http://2.om/> , pense no alinhamento que
> existe entre o circuncentro, ortocentro e baricentro... .
>
> Abraços
>
> Carlos  Victor
>
>
> Em 13 de fevereiro de 2014 13:13, Luís  escreveu:
>
>  Sauda,c~oes,
>
> Como provar a relação abaixo?
>
> R^2=(BC^2+AH^2)/4
>
> Imaginei colocar os pontos B,C,H com as seguintes coordenadas:
>
> B=(0,0)  C=(a,0)  H=(h,y_H) A=(h,y_A)
>
> Daí a gente obtém o ponto H_c=(x,y) com régua e compasso e
> em seguida o ponto A. O circuncentro (O) é calculado e finalmente R.
>
> As contas não são legais com papel e lápis. Alguém poderia dar as
> coordenadas dos pontos A e (O) usando um programa de
> cálculo simbólico ?
>
> Obrigado.
>
> Luís
>
>
> --
> Esta mensagem foi verificada pelo sistema de antivírus e
> acredita-se estar livre de perigo.
>
>
>
>
> --
> Esta mensagem foi verificada pelo sistema de antivírus e
> acredita-se estar livre de perigo.
>
>
>
> --
> Esta mensagem foi verificada pelo sistema de antivírus e
> acredita-se estar livre de perigo.
>
>
>
> --
> Esta mensagem foi verificada pelo sistema de antivírus e
> acredita-se estar livre de perigo.
>
>
>
> --
> Esta mensagem foi verificada pelo sistema de antivírus e
> acredita-se estar livre de perigo.
>

-- 
Esta mensagem foi verificada pelo sistema de antivírus e
 acredita-se estar livre de perigo.



Re: [obm-l] R^2=(BC^2+AH^2)/4

2014-02-17 Por tôpico Carlos Victor
Oi Luís,
Apesar do enunciado não falar, H é o ortocentro do triângulo, ok ?

Abraços

Carlos Victor


Em 16 de fevereiro de 2014 22:33, luiz silva
escreveu:

> AH é a altura relativa à BC?
>
>
>   Em Sábado, 15 de Fevereiro de 2014 17:30, Carlos Victor <
> victorcar...@globo.com> escreveu:
>  Oi Luís,
> digitei errado.
>
> Onde está AM lê-se  AH, ok ?
>
> Desculpe  o engano...
>
> Carlos  Victor
>
>
> Em 15 de fevereiro de 2014 16:53, Carlos Victor 
> escreveu:
>
> Oi Luís,
>
> Seja M o ponto médio de BC  e "O" o circuncentro do triângulo ABC. Prove
> inicialmente que AM= 2.OM <http://2.om/> e aplique Pitágoras no triângulo
> OMC, por exemplo.
> Daí sai legal a relação que tu queres, ok ?
>
> Para provar que AM = 2.OM <http://2.om/> , pense no alinhamento que
> existe entre o circuncentro, ortocentro e baricentro... .
>
> Abraços
>
> Carlos  Victor
>
>
> Em 13 de fevereiro de 2014 13:13, Luís  escreveu:
>
>  Sauda,c~oes,
>
> Como provar a relação abaixo?
>
> R^2=(BC^2+AH^2)/4
>
> Imaginei colocar os pontos B,C,H com as seguintes coordenadas:
>
> B=(0,0)  C=(a,0)  H=(h,y_H) A=(h,y_A)
>
> Daí a gente obtém o ponto H_c=(x,y) com régua e compasso e
> em seguida o ponto A. O circuncentro (O) é calculado e finalmente R.
>
> As contas não são legais com papel e lápis. Alguém poderia dar as
> coordenadas dos pontos A e (O) usando um programa de
> cálculo simbólico ?
>
> Obrigado.
>
> Luís
>
>
> --
> Esta mensagem foi verificada pelo sistema de antivírus e
> acredita-se estar livre de perigo.
>
>
>
>
> --
> Esta mensagem foi verificada pelo sistema de antivírus e
> acredita-se estar livre de perigo.
>
>
>
> --
> Esta mensagem foi verificada pelo sistema de antivírus e
> acredita-se estar livre de perigo.
>

-- 
Esta mensagem foi verificada pelo sistema de antivírus e
 acredita-se estar livre de perigo.



Re: [obm-l] R^2=(BC^2+AH^2)/4

2014-02-15 Por tôpico Carlos Victor
Oi Luís,
digitei errado.

Onde está AM lê-se  AH, ok ?

Desculpe  o engano...

Carlos  Victor


Em 15 de fevereiro de 2014 16:53, Carlos Victor
escreveu:

> Oi Luís,
>
> Seja M o ponto médio de BC  e "O" o circuncentro do triângulo ABC. Prove
> inicialmente que AM= 2.OM e aplique Pitágoras no triângulo OMC, por
> exemplo.
> Daí sai legal a relação que tu queres, ok ?
>
> Para provar que AM = 2.OM , pense no alinhamento que existe entre o
> circuncentro, ortocentro e baricentro... .
>
> Abraços
>
> Carlos  Victor
>
>
> Em 13 de fevereiro de 2014 13:13, Luís  escreveu:
>
>  Sauda,c~oes,
>>
>> Como provar a relação abaixo?
>>
>> R^2=(BC^2+AH^2)/4
>>
>> Imaginei colocar os pontos B,C,H com as seguintes coordenadas:
>>
>> B=(0,0)  C=(a,0)  H=(h,y_H) A=(h,y_A)
>>
>> Daí a gente obtém o ponto H_c=(x,y) com régua e compasso e
>> em seguida o ponto A. O circuncentro (O) é calculado e finalmente R.
>>
>> As contas não são legais com papel e lápis. Alguém poderia dar as
>> coordenadas dos pontos A e (O) usando um programa de
>> cálculo simbólico ?
>>
>> Obrigado.
>>
>> Luís
>>
>>
>> --
>> Esta mensagem foi verificada pelo sistema de antivírus e
>> acredita-se estar livre de perigo.
>>
>
>

-- 
Esta mensagem foi verificada pelo sistema de antivírus e
 acredita-se estar livre de perigo.



Re: [obm-l] R^2=(BC^2+AH^2)/4

2014-02-15 Por tôpico Carlos Victor
Oi Luís,

Seja M o ponto médio de BC  e "O" o circuncentro do triângulo ABC. Prove
inicialmente que AM= 2.OM e aplique Pitágoras no triângulo OMC, por exemplo.
Daí sai legal a relação que tu queres, ok ?

Para provar que AM = 2.OM , pense no alinhamento que existe entre o
circuncentro, ortocentro e baricentro... .

Abraços

Carlos  Victor


Em 13 de fevereiro de 2014 13:13, Luís  escreveu:

> Sauda,c~oes,
>
> Como provar a relação abaixo?
>
> R^2=(BC^2+AH^2)/4
>
> Imaginei colocar os pontos B,C,H com as seguintes coordenadas:
>
> B=(0,0)  C=(a,0)  H=(h,y_H) A=(h,y_A)
>
> Daí a gente obtém o ponto H_c=(x,y) com régua e compasso e
> em seguida o ponto A. O circuncentro (O) é calculado e finalmente R.
>
> As contas não são legais com papel e lápis. Alguém poderia dar as
> coordenadas dos pontos A e (O) usando um programa de
> cálculo simbólico ?
>
> Obrigado.
>
> Luís
>
>
> --
> Esta mensagem foi verificada pelo sistema de antivírus e
> acredita-se estar livre de perigo.
>

-- 
Esta mensagem foi verificada pelo sistema de antivírus e
 acredita-se estar livre de perigo.



Re: [obm-l] Apostila de Desenho 2 Impacto OFF TOPIC

2013-12-03 Por tôpico Carlos Victor
Obrigado João,

Envie-me a sua conta bancária para depósito, ok ?

Também posso lhe enviar o custo pelo correio.

Agradeço e fico a disposição para o que precisares.

Abraços

Carlos Victor


Em 1 de dezembro de 2013 17:54,  escreveu:

>  Senhores:
>
> Ontem (sábado), por volta das 15h em Campo Grande, foi enviada cópia da
> Apostila 2 de Desenho do IMPACTO, uma ao senhor Carlos Victor (Nilópolis -
> RJ), e outra a Graciliano Antônio Damazo (Penápolis - SP).
>
> ATT.
> João (Campo Grande - MS)
>
> --
> Esta mensagem foi verificada pelo sistema de antivírus e
> acredita-se estar livre de perigo.
>

-- 
Esta mensagem foi verificada pelo sistema de antivírus e
 acredita-se estar livre de perigo.



Re: [obm-l] Como que faz??

2013-09-27 Por tôpico Carlos Victor
Olá Eduardo ,
Observe que na circunferência  C´ , o arco CE é dado por : 2x+2y ; pois AC
é tangente  em  à C´ já que o ângulo externo em C no triângulo ACE é dado
por : x+ y . O ângulo CGE é inscrito na circunferência  C   , ok ? . Note
que o ângulo ADE = x+y , está inscrito na circunferência C´´ .

Abs
Carlos  Victor


Em 26 de setembro de 2013 01:05, Eduardo Wilner
escreveu:

> Oi Carlos.
>
> No item 2) vc. diz que  poderia explicar?
>
> Obrigado
>
> [ ]'s
>
>
>   ------
>  *De:* Carlos Victor 
> *Para:* obm-l@mat.puc-rio.br
> *Enviadas:* Terça-feira, 24 de Setembro de 2013 19:30
> *Assunto:* Re: [obm-l] Como que faz??
>
> Olá Douglas,
> Acredito ter conseguido uma resolução para o problema 2 de geometria que
> vc postou aqui  .
>
> Vamos lá  e acompanhe fazendo a figura , ok ?
> vamos provar que na verdade o ângulo DEF é o dobro de ADC.
> Seja  o ângulo  ADC = x e o ângulo CDE = y .
>
> 1) Trace CE e observe que o quadrilátero ACED é inscritível . então AEC =
> x  e  EAC = y .
>
> 2) seja G a intersecção de CD com a circunferência C´ . Trace  EG e
> observe que  o ângulo CGE = x + y . Daí concluímos que o ângulo GED = x .
>
> 3) Não é difícil de mostrar que EB  é bissetriz de AEG . Seja então os
> ângulos AEB= DEB = z .
>
> 4) Trace agora a perpendicular de B  ao segmento ED e seja H o pé desta
> perpendicular. Observe que o quadrilátero BFHD é inscritível , então BHF =
> x .Trace FH e observe que EG é perpendicular  a FH . Seja J a intersecção
> de FH com EG .
>
> 5) Como o triângulo CEG está inscrito na circunferência C´ e observando
> que BF é perpendicular ao lado CD , pelo enunciado ; teremos pela reta de
> SIMSON , que  os pés das  perpendiculares traçadas de B aos lados CG , EG e
> EC  estão alinhados. Sejam então I o pé da perpendicular traçada de ao lado
> EG  e R o pé  da perpendicular traçada de B ao lado CE .
>
> 6) observando os quadriláteros inscritíveis : BIER , BIHE , teremos q
>
>
>
>
>
>
>
>
>
> Em 23 de agosto de 2013 16:03, escreveu:
>
> **
> Olá , alguns alunos do ensino médio da instituição onde trabalho me deram
> alguns problemas do site https://brilliant.org/
> E não consegui achar solução para dois deles, vou escreve-los abaixo e se
> alguém puder me ajudar agradeço.
>
> PROBLEMA 1: Dada uma função f:R->R tal que f(2x^2 -1)=2(f(x))^2 -1 e f(x)
> é um polinômio de grau 13, sendo assim determine o coeficiente de x^5 de
> f(x).
>
> PROBLEMA 2: Seja uma circunferência C'  e um ponto externo A , traça-se
> por A duas tangentes a circunferência que a interceptam nos pontos B e C ,
> marca-se no prolongamento de AB no sentido de A para B um ponto D tal que o
> ângulo ADC=25 graus, traça-se por B uma perpendicular ao segmento CD que
> intercepta CD em F . Agora considere um outra circunferência C''
> circunscrita ao triângulo ADC que intercepta a primeira circunferência C'
> no ponto E . Determinar a medida do ângulo DEF.
>
>
> Obs: Fiz a segunda figura no geogebra e encontrei 50 graus como resposta ,
> preciso na verdade de uma resolução.
>
> Att, Douglas Oliveira.
>
>
> --
> Esta mensagem foi verificada pelo sistema de antivírus e
> acredita-se estar livre de perigo.
>
>
>
> --
> Esta mensagem foi verificada pelo sistema de antivírus e
> acredita-se estar livre de perigo.
>
>
>
> --
> Esta mensagem foi verificada pelo sistema de antivírus e
> acredita-se estar livre de perigo.
>

-- 
Esta mensagem foi verificada pelo sistema de antivírus e
 acredita-se estar livre de perigo.



Re: [obm-l] Como que faz??

2013-09-24 Por tôpico Carlos Victor
Olá ,
sem querer dei um enter e a mensagem foi enviada incompleta , também
digitei um ângulo errado no ítem (3) : acompanhe agora :


completando o ítem (6)  e consertando que  no ítem (3) AEB= DEG = z , pois
eu digitei DEB.
teremos que o ângulo BHI = z ; ânguloFIG = 90-x-z  e ângulo EIH = 90-x-z .

7) concluímos então que IJ é bissetriz e altura no triângulo FIH ;
ou seja FJ =JH , daí o triângulo FEH é isósceles .

Temos então que  ângulo FEJ = x ; ou seja

ângulo FED = 2x ... Ufa !!!

Abraços

Carlos Victor



Em 24 de setembro de 2013 19:30, Carlos Victor escreveu:

> Olá Douglas,
> Acredito ter conseguido uma resolução para o problema 2 de geometria que
> vc postou aqui  .
>
> Vamos lá  e acompanhe fazendo a figura , ok ?
> vamos provar que na verdade o ângulo DEF é o dobro de ADC.
> Seja  o ângulo  ADC = x e o ângulo CDE = y .
>
> 1) Trace CE e observe que o quadrilátero ACED é inscritível . então AEC =
> x  e  EAC = y .
>
> 2) seja G a intersecção de CD com a circunferência C´ . Trace  EG e
> observe que  o ângulo CGE = x + y . Daí concluímos que o ângulo GED = x .
>
> 3) Não é difícil de mostrar que EB  é bissetriz de AEG . Seja então os
> ângulos AEB= DEB = z .
>
> 4) Trace agora a perpendicular de B  ao segmento ED e seja H o pé desta
> perpendicular. Observe que o quadrilátero BFHD é inscritível , então BHF =
> x .Trace FH e observe que EG é perpendicular  a FH . Seja J a intersecção
> de FH com EG .
>
> 5) Como o triângulo CEG está inscrito na circunferência C´ e observando
> que BF é perpendicular ao lado CD , pelo enunciado ; teremos pela reta de
> SIMSON , que  os pés das  perpendiculares traçadas de B aos lados CG , EG e
> EC  estão alinhados. Sejam então I o pé da perpendicular traçada de ao lado
> EG  e R o pé  da perpendicular traçada de B ao lado CE .
>
> 6) observando os quadriláteros inscritíveis : BIER , BIHE , teremos q
>
>
>
>
>
>
>
>
>
> Em 23 de agosto de 2013 16:03, escreveu:
>
> **
>>
>> Olá , alguns alunos do ensino médio da instituição onde trabalho me deram
>> alguns problemas do site https://brilliant.org/
>>
>> E não consegui achar solução para dois deles, vou escreve-los abaixo e se
>> alguém puder me ajudar agradeço.
>>
>>
>>
>> PROBLEMA 1: Dada uma função f:R->R tal que f(2x^2 -1)=2(f(x))^2 -1 e f(x)
>> é um polinômio de grau 13, sendo assim determine o coeficiente de x^5 de
>> f(x).
>>
>>
>>
>> PROBLEMA 2: Seja uma circunferência C'  e um ponto externo A , traça-se
>> por A duas tangentes a circunferência que a interceptam nos pontos B e C ,
>> marca-se no prolongamento de AB no sentido de A para B um ponto D tal que o
>> ângulo ADC=25 graus, traça-se por B uma perpendicular ao segmento CD que
>> intercepta CD em F . Agora considere um outra circunferência C''
>> circunscrita ao triângulo ADC que intercepta a primeira circunferência C'
>> no ponto E . Determinar a medida do ângulo DEF.
>>
>>
>>
>>
>>
>> Obs: Fiz a segunda figura no geogebra e encontrei 50 graus como resposta
>> , preciso na verdade de uma resolução.
>>
>>
>>
>> Att, Douglas Oliveira.
>>
>>
>> --
>> Esta mensagem foi verificada pelo sistema de antivírus e
>> acredita-se estar livre de perigo.
>>
>
>

-- 
Esta mensagem foi verificada pelo sistema de antivírus e
 acredita-se estar livre de perigo.



Re: [obm-l] Como que faz??

2013-09-24 Por tôpico Carlos Victor
Olá Douglas,
Acredito ter conseguido uma resolução para o problema 2 de geometria que vc
postou aqui  .

Vamos lá  e acompanhe fazendo a figura , ok ?
vamos provar que na verdade o ângulo DEF é o dobro de ADC.
Seja  o ângulo  ADC = x e o ângulo CDE = y .

1) Trace CE e observe que o quadrilátero ACED é inscritível . então AEC = x
 e  EAC = y .

2) seja G a intersecção de CD com a circunferência C´ . Trace  EG e observe
que  o ângulo CGE = x + y . Daí concluímos que o ângulo GED = x .

3) Não é difícil de mostrar que EB  é bissetriz de AEG . Seja então os
ângulos AEB= DEB = z .

4) Trace agora a perpendicular de B  ao segmento ED e seja H o pé desta
perpendicular. Observe que o quadrilátero BFHD é inscritível , então BHF =
x .Trace FH e observe que EG é perpendicular  a FH . Seja J a intersecção
de FH com EG .

5) Como o triângulo CEG está inscrito na circunferência C´ e observando que
BF é perpendicular ao lado CD , pelo enunciado ; teremos pela reta de
SIMSON , que  os pés das  perpendiculares traçadas de B aos lados CG , EG e
EC  estão alinhados. Sejam então I o pé da perpendicular traçada de ao lado
EG  e R o pé  da perpendicular traçada de B ao lado CE .

6) observando os quadriláteros inscritíveis : BIER , BIHE , teremos q









Em 23 de agosto de 2013 16:03, escreveu:

> **
>
> Olá , alguns alunos do ensino médio da instituição onde trabalho me deram
> alguns problemas do site https://brilliant.org/
>
> E não consegui achar solução para dois deles, vou escreve-los abaixo e se
> alguém puder me ajudar agradeço.
>
>
>
> PROBLEMA 1: Dada uma função f:R->R tal que f(2x^2 -1)=2(f(x))^2 -1 e f(x)
> é um polinômio de grau 13, sendo assim determine o coeficiente de x^5 de
> f(x).
>
>
>
> PROBLEMA 2: Seja uma circunferência C'  e um ponto externo A , traça-se
> por A duas tangentes a circunferência que a interceptam nos pontos B e C ,
> marca-se no prolongamento de AB no sentido de A para B um ponto D tal que o
> ângulo ADC=25 graus, traça-se por B uma perpendicular ao segmento CD que
> intercepta CD em F . Agora considere um outra circunferência C''
> circunscrita ao triângulo ADC que intercepta a primeira circunferência C'
> no ponto E . Determinar a medida do ângulo DEF.
>
>
>
>
>
> Obs: Fiz a segunda figura no geogebra e encontrei 50 graus como resposta ,
> preciso na verdade de uma resolução.
>
>
>
> Att, Douglas Oliveira.
>
>
> --
> Esta mensagem foi verificada pelo sistema de antivírus e
> acredita-se estar livre de perigo.
>

-- 
Esta mensagem foi verificada pelo sistema de antivírus e
 acredita-se estar livre de perigo.



[obm-l] Re: [obm-l] Álgebra(não tá saindo)

2013-09-15 Por tôpico Carlos Victor
Olá Marcone,

Na hipótese de que quatro vezes maior significa o quádruplo , teremos :

Seja N = y..y6, o número procurado, em que y representa algarismos não
necessariamente iguais . Podemos escrever  N = 10X + 6 .

Logo  4N = 6.(10^n) + X  = 6.( 10^n) + ( N -6)/10 ; ou seja ,

N = 2( 10^(n+1) -1)/13.

Como  10^3 = -1(mod13) , então o menor  N = 2(10^6-1)/13 = 153846 .

Abraços

Carlos  Victor


Em 14 de setembro de 2013 19:15, marcone augusto araújo borges <
marconeborge...@hotmail.com> escreveu:

> Encontre o menor inteiro positivo n que possui as seguintes propriedades:
> I. Em sua representação tem o 6 como último dígito
> II.Se o último dígito(6) é apagado  e colocado na frente dos dígitos
> restantes,o número resultante
> é quatro vezes maior que o número original n
>
> --
> Esta mensagem foi verificada pelo sistema de antivírus e
> acredita-se estar livre de perigo.
>

-- 
Esta mensagem foi verificada pelo sistema de antivírus e
 acredita-se estar livre de perigo.



Re: [obm-l] trigonometria

2013-08-05 Por tôpico Carlos Victor
Olá  João ,

Esta questão é  de uma olimpíada não brasileira ou de um livro de
olimpíadas ( não lembro qual País), mas encontrar os outros ângulos é um
trabalho árduo e há uma estratégia para a sua solução geométrica . A que
conheço ( em que o mestre Antonio Luis( Gandhi) me mostrou)  é traçar os
simétricos de D e E em relação à  BD e CE , respectivamente, sobre BC .
Faça  uma análise nos triângulos que surgirão , no sentido de que a
bissetriz interna e externa de um triângulo se encontram num ex-incentro e,
aparecerá um ângulo de 120º que é o mentor da solução, ok ? Vale apena
pensar nessa solução ...

Abraços

Carlos Victor


Em 5 de agosto de 2013 11:04, Nehab  escreveu:

>  Ora João!
>
> Nem vem. Você é muito inteligente para odiar Geometria...
> Não acho má ideia você estudar um pouquinho disso...
> Costumo ter sucesso ensinando essa parte maravilhosa da Matemática para
> quem odeia Geometria (hahaha) e gosta de Trigonometria...
> Veja que o ângulo A é imediato... Chamando de I o incentro, segue-se:
> a) No triângulo BIC, ang(BIC) = 180 - (B/2+C/2) = 90 + A/2
> b) No triângulo EID, ang(EID) = 180 - (24 + 18) = 138
> c) Mas ang(BIC) = ang(EID) e daí sai A: 90 + A/2 = 138, ou seja, A = 96
>
> Tente completar a solução...
>
> Grande abraço,
> Nehab
>
>
> On 04/08/2013 23:37, João Maldonado wrote:
>
> Fala professor!
>
> Adorei a resolução, tinha esquecido do 4sen18.cos36 =1   =D
> Na verdade o problema era de geometria, mas como eu sou péssimo em GP,
> sempre resolvo tudo por trigonometria (meu professor fala que eu sou louco)
> O problema era o seguinte:
> Em um triângulo ABC, D e E são os pés das bissetrizes traçadas dos
> vértices B e C respectivamente. CED = 24 graus e BDE = 18 graus, calcule os
> ângulos do triângulo.
>
> De acordo com o que foi dito os ângulos são 2*36 = 72 graus, 12 graus e 96
> graus
>
> []'s
> João
>
>  --
> Date: Sat, 3 Aug 2013 23:16:56 -0300
> From: carlos.ne...@gmail.com
> To: obm-l@mat.puc-rio.br
> Subject: Re: [obm-l] trigonometria
>
> Caramba, João,
> Gostei. Espertinho! Meu raciocínio navegou assim:
>
> a) 66 = 36 + 30, então 36  é um angulo duplamente interessante pro
> problema.
>
> b) O que eu sei sobre 36 e companhia? Que o sen18 gosta do cos36 pois
> 4sen18.cos36 =1.
> Isso não é exatamente um coelho da cartola, pois essa igualdade é
> clássica  se você estudou os triângulos isósceles que possuem um ângulo de
> 36 (trace as diagonais de um pentágono e está tudo lá). Nesses triângulos o
> lado maior é phi vezes o lado menor, ou seja, phi = (raiz(5) + 1)/2 vezes o
> lado menor (uma semelhançazinha). Além disso, esse phi é adorável e é
> manjada razão áurea.
> Dai é fácil você ver nos triângulos isósceles citados (trace as alturas
> deles) que sen18 = 1/2phi e cos36 = phi/2.
> Logo, 4sen18.cos36 = 1...
>
> c) Assim, achei que seria legal encostar um cos36 no lado direito...
>
> Então, fica assim:
>
> tgx = tg 66 - 2sen18/cos66 = [ sen66 - 2sen18] / cos66
> tgx. cos36 = B/C onde
> B = [2sen66cos36 - *4sen18cos36** *] e
> C = 2cos66
> Desenvolvendo B, vem:
> B = sen30 + sen102 - *1* =
> B = sen102 - sen 30 (passagem boba e bonita, né)
> B = 2sen36cos66
> Dai tgx.cos36 = B/C = sen36.
> Logo, x = 36 (se não foi dito que x está entre 0 e 180, então x = 36 +
> k180)
>
> Abraços
> Nehab
>
> On 03/08/2013 18:08, João Maldonado wrote:
>
> tgx = tg66 - 2sen18/cos66
> Como achar x?
>
>
>
> --
> Esta mensagem foi verificada pelo sistema de antivírus e
> acredita-se estar livre de perigo.
>
> --
> Esta mensagem foi verificada pelo sistema de antivírus e
> acredita-se estar livre de perigo.
>
>
>
> --
> Esta mensagem foi verificada pelo sistema de antivírus e
> acredita-se estar livre de perigo.
>

-- 
Esta mensagem foi verificada pelo sistema de antivírus e
 acredita-se estar livre de perigo.



Re: [obm-l] trigonometria

2013-08-04 Por tôpico Carlos Victor
Desculpem ,

digitei errado na linha
(1/2)cosy +seny.cos30 =(1/2).cos30 +2sen18.sen36.seny

que na verdade é
(1/2)cosy +seny.cos30 =(1/2).cosy +2sen18.sen36.seny .

Abraços

Carlos  Victor



Em 4 de agosto de 2013 14:09, Carlos Victor escreveu:

> Olá Marcone, aproveitando a ideia do meu mestre Nehab, podemos escrever :
>
> acertando a expressão dada chegamos a
>
> sen(66-x) = 2sen18.cosx
>
> tomando 36-x =y ,teremos
>
> sen(30+y) = 2sen18.cos(36-y) = 2sen18[ cos36.cosy + sen36.seny]
>
> sen(30+y) = 2sen18.cos36.cosy + 2sen18.sen36.seny
>
> sen30.cosy+seny.cos30 = 2sen18.cos36.cosy + 2sen18.sen36.seny
>
> usando que o Nehab lembrou , teremos
>
> (1/2)cosy +seny.cos30 =(1/2).cos30 +2sen18.sen36.seny
>
> seny.cos30 = 2sen18.sen36.seny .
>
> Não é difícil de mostrar que 2sen18.sen36 é diferente de cos30 ;
>
> logo devemos ter seny =0 ; ou seja y = k180 ; daí
>
> x= k180 +36 .
>
> Agradecendo ao Nehab ,
>
> Abraços
>
> Carlos Victor
>
>
> Em 4 de agosto de 2013 13:33, marcone augusto araújo borges <
> marconeborge...@hotmail.com> escreveu:
>
>  Essa foi muito legal.
>>
>> --
>> From: ilhadepaqu...@bol.com.br
>> To: obm-l@mat.puc-rio.br
>> Subject: Re: [obm-l] trigonometria
>> Date: Sun, 4 Aug 2013 10:59:12 -0300
>>
>>
>> correção
>> 2014-1974=40 bodas de rubi ou esmeralda, mas mesmo assim merece uma
>> comemoração
>> Abraço a todos
>> Hermann
>>
>> - Original Message -
>> *From:* Carlos Victor 
>> *To:* obm-l@mat.puc-rio.br
>> *Sent:* Sunday, August 04, 2013 9:32 AM
>> *Subject:* Re: [obm-l] trigonometria
>>
>> Olá grande Mestre Nehab,
>> Você me ensinou também em 1974 que poderíamos retirar a
>> igualdade 4sen18.cos36 =1, fazendo :
>>
>> sen18.cos36 = sen18.cos18.cos36/cos18 = sen36.cos36./2cos18=
>>
>> sen72/4cos18 = 1/4 .( saudades!!)
>>
>> Abraços
>>
>> Carlos Victor
>>
>>
>> Em 3 de agosto de 2013 23:16, Nehab  escreveu:
>>
>>  Caramba, João,
>> Gostei. Espertinho! Meu raciocínio navegou assim:
>>
>> a) 66 = 36 + 30, então 36  é um angulo duplamente interessante pro
>> problema.
>>
>> b) O que eu sei sobre 36 e companhia? Que o sen18 gosta do cos36 pois
>> 4sen18.cos36 =1.
>> Isso não é exatamente um coelho da cartola, pois essa igualdade é
>> clássica  se você estudou os triângulos isósceles que possuem um ângulo de
>> 36 (trace as diagonais de um pentágono e está tudo lá). Nesses triângulos o
>> lado maior é phi vezes o lado menor, ou seja, phi = (raiz(5) + 1)/2 vezes o
>> lado menor (uma semelhançazinha). Além disso, esse phi é adorável e é
>> manjada razão áurea.
>> Dai é fácil você ver nos triângulos isósceles citados (trace as alturas
>> deles) que sen18 = 1/2phi e cos36 = phi/2.
>> Logo, 4sen18.cos36 = 1...
>>
>> c) Assim, achei que seria legal encostar um cos36 no lado direito...
>>
>> Então, fica assim:
>>
>> tgx = tg 66 - 2sen18/cos66 = [ sen66 - 2sen18] / cos66
>> tgx. cos36 = B/C onde
>> B = [2sen66cos36 - *4sen18cos36** *] e
>> C = 2cos66
>> Desenvolvendo B, vem:
>> B = sen30 + sen102 - *1* =
>> B = sen102 - sen 30 (passagem boba e bonita, né)
>> B = 2sen36cos66
>> Dai tgx.cos36 = B/C = sen36.
>> Logo, x = 36 (se não foi dito que x está entre 0 e 180, então x = 36 +
>> k180)
>>
>> Abraços
>> Nehab
>>
>>
>> On 03/08/2013 18:08, João Maldonado wrote:
>>
>> tgx = tg66 - 2sen18/cos66
>> Como achar x?
>>
>>
>>
>> --
>> Esta mensagem foi verificada pelo sistema de antivírus e
>> acredita-se estar livre de perigo.
>>
>>
>>
>> --
>> Esta mensagem foi verificada pelo sistema de antivírus e
>> acredita-se estar livre de perigo.
>>
>>
>> --
>> Esta mensagem foi verificada pelo sistema de antivírus e
>> acredita-se estar livre de perigo.
>>
>> --
>> Esta mensagem foi verificada pelo sistema de antivírus e
>> acredita-se estar livre de perigo.
>>
>
>

-- 
Esta mensagem foi verificada pelo sistema de antivírus e
 acredita-se estar livre de perigo.



Re: [obm-l] trigonometria

2013-08-04 Por tôpico Carlos Victor
Olá Marcone, aproveitando a ideia do meu mestre Nehab, podemos escrever :

acertando a expressão dada chegamos a

sen(66-x) = 2sen18.cosx

tomando 36-x =y ,teremos

sen(30+y) = 2sen18.cos(36-y) = 2sen18[ cos36.cosy + sen36.seny]

sen(30+y) = 2sen18.cos36.cosy + 2sen18.sen36.seny

sen30.cosy+seny.cos30 = 2sen18.cos36.cosy + 2sen18.sen36.seny

usando que o Nehab lembrou , teremos

(1/2)cosy +seny.cos30 =(1/2).cos30 +2sen18.sen36.seny

seny.cos30 = 2sen18.sen36.seny .

Não é difícil de mostrar que 2sen18.sen36 é diferente de cos30 ;

logo devemos ter seny =0 ; ou seja y = k180 ; daí

x= k180 +36 .

Agradecendo ao Nehab ,

Abraços

Carlos Victor


Em 4 de agosto de 2013 13:33, marcone augusto araújo borges <
marconeborge...@hotmail.com> escreveu:

> Essa foi muito legal.
>
> --
> From: ilhadepaqu...@bol.com.br
> To: obm-l@mat.puc-rio.br
> Subject: Re: [obm-l] trigonometria
> Date: Sun, 4 Aug 2013 10:59:12 -0300
>
>
> correção
> 2014-1974=40 bodas de rubi ou esmeralda, mas mesmo assim merece uma
> comemoração
> Abraço a todos
> Hermann
>
> - Original Message -
> *From:* Carlos Victor 
> *To:* obm-l@mat.puc-rio.br
> *Sent:* Sunday, August 04, 2013 9:32 AM
> *Subject:* Re: [obm-l] trigonometria
>
> Olá grande Mestre Nehab,
> Você me ensinou também em 1974 que poderíamos retirar a
> igualdade 4sen18.cos36 =1, fazendo :
>
> sen18.cos36 = sen18.cos18.cos36/cos18 = sen36.cos36./2cos18=
>
> sen72/4cos18 = 1/4 .( saudades!!)
>
> Abraços
>
> Carlos Victor
>
>
> Em 3 de agosto de 2013 23:16, Nehab  escreveu:
>
>  Caramba, João,
> Gostei. Espertinho! Meu raciocínio navegou assim:
>
> a) 66 = 36 + 30, então 36  é um angulo duplamente interessante pro
> problema.
>
> b) O que eu sei sobre 36 e companhia? Que o sen18 gosta do cos36 pois
> 4sen18.cos36 =1.
> Isso não é exatamente um coelho da cartola, pois essa igualdade é
> clássica  se você estudou os triângulos isósceles que possuem um ângulo de
> 36 (trace as diagonais de um pentágono e está tudo lá). Nesses triângulos o
> lado maior é phi vezes o lado menor, ou seja, phi = (raiz(5) + 1)/2 vezes o
> lado menor (uma semelhançazinha). Além disso, esse phi é adorável e é
> manjada razão áurea.
> Dai é fácil você ver nos triângulos isósceles citados (trace as alturas
> deles) que sen18 = 1/2phi e cos36 = phi/2.
> Logo, 4sen18.cos36 = 1...
>
> c) Assim, achei que seria legal encostar um cos36 no lado direito...
>
> Então, fica assim:
>
> tgx = tg 66 - 2sen18/cos66 = [ sen66 - 2sen18] / cos66
> tgx. cos36 = B/C onde
> B = [2sen66cos36 - *4sen18cos36** *] e
> C = 2cos66
> Desenvolvendo B, vem:
> B = sen30 + sen102 - *1* =
> B = sen102 - sen 30 (passagem boba e bonita, né)
> B = 2sen36cos66
> Dai tgx.cos36 = B/C = sen36.
> Logo, x = 36 (se não foi dito que x está entre 0 e 180, então x = 36 +
> k180)
>
> Abraços
> Nehab
>
>
> On 03/08/2013 18:08, João Maldonado wrote:
>
> tgx = tg66 - 2sen18/cos66
> Como achar x?
>
>
>
> --
> Esta mensagem foi verificada pelo sistema de antivírus e
> acredita-se estar livre de perigo.
>
>
>
> --
> Esta mensagem foi verificada pelo sistema de antivírus e
> acredita-se estar livre de perigo.
>
>
> --
> Esta mensagem foi verificada pelo sistema de antivírus e
> acredita-se estar livre de perigo.
>
> --
> Esta mensagem foi verificada pelo sistema de antivírus e
> acredita-se estar livre de perigo.
>

-- 
Esta mensagem foi verificada pelo sistema de antivírus e
 acredita-se estar livre de perigo.



Re: [obm-l] trigonometria

2013-08-04 Por tôpico Carlos Victor
Olá grande Mestre Nehab,
Você me ensinou também em 1974 que poderíamos retirar a
igualdade 4sen18.cos36 =1, fazendo :

sen18.cos36 = sen18.cos18.cos36/cos18 = sen36.cos36./2cos18=

sen72/4cos18 = 1/4 .( saudades!!)

Abraços

Carlos Victor


Em 3 de agosto de 2013 23:16, Nehab  escreveu:

>  Caramba, João,
> Gostei. Espertinho! Meu raciocínio navegou assim:
>
> a) 66 = 36 + 30, então 36  é um angulo duplamente interessante pro
> problema.
>
> b) O que eu sei sobre 36 e companhia? Que o sen18 gosta do cos36 pois
> 4sen18.cos36 =1.
> Isso não é exatamente um coelho da cartola, pois essa igualdade é
> clássica  se você estudou os triângulos isósceles que possuem um ângulo de
> 36 (trace as diagonais de um pentágono e está tudo lá). Nesses triângulos o
> lado maior é phi vezes o lado menor, ou seja, phi = (raiz(5) + 1)/2 vezes o
> lado menor (uma semelhançazinha). Além disso, esse phi é adorável e é
> manjada razão áurea.
> Dai é fácil você ver nos triângulos isósceles citados (trace as alturas
> deles) que sen18 = 1/2phi e cos36 = phi/2.
> Logo, 4sen18.cos36 = 1...
>
> c) Assim, achei que seria legal encostar um cos36 no lado direito...
>
> Então, fica assim:
>
> tgx = tg 66 - 2sen18/cos66 = [ sen66 - 2sen18] / cos66
> tgx. cos36 = B/C onde
> B = [2sen66cos36 - *4sen18cos36** *] e
> C = 2cos66
> Desenvolvendo B, vem:
> B = sen30 + sen102 - *1* =
> B = sen102 - sen 30 (passagem boba e bonita, né)
> B = 2sen36cos66
> Dai tgx.cos36 = B/C = sen36.
> Logo, x = 36 (se não foi dito que x está entre 0 e 180, então x = 36 +
> k180)
>
> Abraços
> Nehab
>
>
> On 03/08/2013 18:08, João Maldonado wrote:
>
> tgx = tg66 - 2sen18/cos66
> Como achar x?
>
>
>
> --
> Esta mensagem foi verificada pelo sistema de antivírus e
> acredita-se estar livre de perigo.
>

-- 
Esta mensagem foi verificada pelo sistema de antivírus e
 acredita-se estar livre de perigo.



[obm-l] Re: [obm-l] Re: [obm-l] RE: [obm-l] Re: [obm-l] Soluções inteiras da equação x/y = x - y

2013-06-18 Por tôpico Carlos Victor
Olá ,
É interessante também  observar que nesses tipos de problemas , já que y=0
e  y =1 não são soluções, podemos escrever :

x = y^2/(y-1) = y+1 +1/(y-1) ; ou seja (y-1) deve ser -1 ou +1 . Daí y = 2
e x = 4 .

Abraços

Carlos Victor



Em 18 de junho de 2013 19:43, Marcelo Salhab Brogliato
escreveu:

> É verdade! Nesse caso, chega-se a mesma conclusão, mas em outros problemas
> esse erro pode "esconder" alguma possível solução.
>
> Obrigado! :)
>
> Abraços,
> Salhab
>
>
> 2013/6/18 Paulo Argolo 
>
>> Caro Salhab,
>>
>> Na verdade:  k|y e y|k => |k| = |y|
>> De qualquer forma, chega-se a mesma conclusão.
>>
>> Um abraço do Paulo Argolo!
>> ___
>>
>>
>>
>> Date: Tue, 18 Jun 2013 15:14:58 -0300
>> Subject: [obm-l] Re: [obm-l] Soluções inteiras da equação x/y = x - y
>> From: msbro...@gmail.com
>> To: obm-l@mat.puc-rio.br
>>
>> Olá, Ennius, tudo bem?
>>
>> Se as soluções são inteiras, então temos que y|x, logo: x = ky. Assim:
>> ky/y = ky - y
>>
>> k = ky - y
>> k + y = ky
>>
>> Então: k|y e y|k => y = k.
>>
>> y + y = y*y => y(y-2) = 0 => y = 0 ou y = 2. Mas y não pode ser 0, pois a
>> equação original é x/y = x - y.
>> Assim: y = 2, k = 2 e x = ky = 4.
>>
>> Abraços,
>> Salhab
>>
>>
>> 2013/6/18 ennius 
>> Colegas da Lista,
>>
>> Como mostrar que a equação x/y = x - y não admite soluções inteiras,
>>  além de x = 4 e y = 2?
>> --
>>
>> --
>> Esta mensagem foi verificada pelo sistema de antivírus e
>>  acredita-se estar livre de perigo.
>>
>> =
>> Instruções para entrar na lista, sair da lista e usar a lista em
>> http://www.mat.puc-rio.br/~obmlistas/obm-l.html
>> =
>>
>>
>> --
>> Esta mensagem foi verificada pelo sistema de antivírus e
>> acredita-se estar livre de perigo.
>> --
>> Esta mensagem foi verificada pelo sistema de antivírus e
>>  acredita-se estar livre de perigo.
>>
>>
>> =
>> Instruções para entrar na lista, sair da lista e usar a lista em
>> http://www.mat.puc-rio.br/~obmlistas/obm-l.html
>> =
>>
>
>
> --
> Esta mensagem foi verificada pelo sistema de antivírus e
> acredita-se estar livre de perigo.
>

-- 
Esta mensagem foi verificada pelo sistema de antivírus e
 acredita-se estar livre de perigo.



[obm-l] Re: [obm-l] Duas perguntas(teoria dos números)

2013-05-27 Por tôpico Carlos Victor
Olá ,
Observando que m+48 = 2^k e m-48 = 2^(n-k) ,
teremos  3 = 2^(k-5) - 2^(n-k-5) ; ou seja k - 5 =2 e n-k-5 = 0 .

Então n =12 . Está Ok isso ?

Carlos Victor




Em 27 de maio de 2013 14:16, marcone augusto araújo borges <
marconeborge...@hotmail.com> escreveu:

> 1) Gostaria de saber se a soma de duas ou mais potencias de base 2
> distintas pode ser uma potencia de base 2.
>
> Acredito que não e escrevendo esses números na base 2 talvez se possa
> mostrar isso.
>
> 2) Desconfio que   2304 + 2^n é um quadrado perfeito para um único valor
> de n.
>
> Eu fiz 2^n = (m + 48)(m - 48)
> m + 48 e m - 48 devem ser potencias de base 2
> As únicas potencias de base 2 cuja diferença é 96 são 128 e 32
> Dai o único valor de n seria 12
> Um esclarecimento seria muito bem vindo
>
> Desde já agradeço
>
>
>


Re: [obm-l] Problema de Geometria

2013-04-28 Por tôpico Carlos Victor
Olá Raphael,
Pense no seguinte :

1) Trace OC
2) Trace BD
3) Conclua que BD é o dobro de OC.
4) Denomine EF = x
5) Faça  a semelhança de OCF com BFD e determine x , ok ?

Abraços

Carlos  Victor


Em 28 de abril de 2013 18:19, Raphael Feijao
escreveu:

> O segmento AB é o diametro de uma circunferencia de centro O. Toma-se um
> ponto C desse círculo e prolonga-se o segmento AC de um segmento CD igual a
> AC. O segmento OD corta a circunferencia em E e corta o segmento BC em F.
> Se AB=a e OD=b. Calcule EF.
>


[obm-l] Re: [obm-l] Potência "encardida"

2013-04-02 Por tôpico Carlos Victor
Olá Vanderlei ,
O que vc pode perceber que  na sequência 2^2, 2^22,2^42,..., todos terminam
em 04 . 2^222 está nesta sequência , ok ?

Abraços

Carlos Victor

Em 2 de abril de 2013 13:01, Vanderlei *  escreveu:

> *Bom dia, pessoal! Gostaria de uma ajuda na seguinte questão, a qual eu
> só consegui com binômio de Newton e alguma força bruta.*
> **
> *Quais são os dois últimos algarismos do resultado de 2^222?*
> **
> *A resposta é 04.*
> **
> *Obrigado!*
> **
> *Vanderlei*
>
> --
> Esta mensagem foi verificada pelo sistema de antivírus e
> acredita-se estar livre de perigo.

-- 
Esta mensagem foi verificada pelo sistema de antivírus e
 acredita-se estar livre de perigo.



Re: [obm-l] Desigualdades

2013-03-23 Por tôpico Carlos Victor
Ok,  Meu Grande Mestre Nehab,

Um Saudoso Abraço

Carlos

Victor

Em 20 de março de 2013 23:21, Nehab  escreveu:

>  Oi, querido amigo,
>
> Apenas uma observação:
> Ficou provado que 96 majora a soma, mas ainda temos que explicitar x, y e
> z com xyz = 32 que faz a soma ser IGUAL a 96.
> Em sua prova a igualdade a 96 valeria se houvesse x, y e z com 4xy = z^2
> (e naturalmente xyz = 32).
> De fato isto ocorre qdo z = 4 e dai, x =4 e y = 2.
>
> Um grande abraço,
> Saudades
> Nehab
>
>
> On 20/03/2013 08:51, Carlos Victor wrote:
>
> Olá ,
> acredito que dê  só por médias :
> 4xy + (x^2 + 4y^2) + 2z^2 >= 4xy + 4xy + 2z^2 >= 3.raiz cúbica de (
> 32(xyz)^2) =3.32 = 96.
>
> Carlos Victor
>
>
> Em 19 de março de 2013 20:41, Bernardo Freitas Paulo da Costa <
> bernardo...@gmail.com> escreveu:
>
>> 2013/3/19 Carlos Yuzo Shine :
>> > Só para evitar derivadas (especialmente de mais de uma variável, em que
>> há vários detalhes), aí vão soluções:
>> >
>> > 1) Pela desigualdade de médias, a expressão é igual a 4xy + (x^2 +
>> 4y^2) + 2z^2 >= 4xy + 4xy + 2z^2 = 8xy + 2z^2 >= 4xyz = 4*32 = 128. A
>> igualdade ocorre quando x = 2y e 4xy = z^2, ou seja, x = 2^(11/6), y =
>> 2^(5/6) e z = 2^(7/3).
>>
>>  Oi Shine,
>>
>> eu não entendi a passagem 8xy + 2z^2 >= 4xyz. Não pode ser só
>> desigualdade das médias, porque essa é homogênea, e todos os termos da
>> esquerda são de ordem dois. Acho que faltou uma dica para o seu caro
>> leitor.
>>
>> Pensando um pouco mais, eu resolveria com multiplicadores de Lagrange
>> (e portanto com derivadas). Mas se fosse antes de aprender Lagrange,
>> eu teria feito assim:
>>
>> Note que se z é fixo, temos que minimizar (x + 2y)^2, com xy =
>> constante. (Aplicando a famosa técnica "escolha produtos notáveis que
>> vão te ajudar".) Pela MA >= MG, obtemos x = 2y (como todo mundo
>> obteve...).
>>
>> xy = 32/z, x = 2y => 2y^2 = 32/z => y^2 = 16/z, x^2 = 4*16/z e
>> portanto x^2 + 4xy + 4y^2 = 4*16/z + 4*32/z + 4*16/z = 4*32*2/z.
>>
>> Queremos minimizar 4*32*2/z + 2z^2. Pela desigualdade das médias com 3
>> termos: 4*32/z + 4*32/z + 2z^2 >= 3 * (4*32 * 4*32 * 2)^1/3 = 3 *
>> (2^(2+5+2+5+1))^1/3 = 3 * 2^5 = 3 * 32 = 96. A igualdade ocorre para
>>
>> 4*32/z = 2z^2 <=> 64 = z^3, ou seja z = 4, y = 4/raiz(z) = 4/2 = 2, x = 4.
>>
>> Verificando: x^2 = 4^2 = 16
>> 4xy = 4*2*4 = 32
>> 4*y^2 = 4*2^2 = 16
>> 2z^2 = 2*4^2 = 32
>> Somando = 96.
>>
>> --
>> Bernardo Freitas Paulo da Costa
>>
>> =
>> Instruções para entrar na lista, sair da lista e usar a lista em
>> http://www.mat.puc-rio.br/~obmlistas/obm-l.html
>> =
>>
>
>
>


Re: [obm-l] Desigualdades

2013-03-20 Por tôpico Carlos Victor
Olá ,
acredito que dê  só por médias :
4xy + (x^2 + 4y^2) + 2z^2 >= 4xy + 4xy + 2z^2 >= 3.raiz cúbica de (
32(xyz)^2) =3.32 = 96.

Carlos Victor


Em 19 de março de 2013 20:41, Bernardo Freitas Paulo da Costa <
bernardo...@gmail.com> escreveu:

> 2013/3/19 Carlos Yuzo Shine :
> > Só para evitar derivadas (especialmente de mais de uma variável, em que
> há vários detalhes), aí vão soluções:
> >
> > 1) Pela desigualdade de médias, a expressão é igual a 4xy + (x^2 + 4y^2)
> + 2z^2 >= 4xy + 4xy + 2z^2 = 8xy + 2z^2 >= 4xyz = 4*32 = 128. A igualdade
> ocorre quando x = 2y e 4xy = z^2, ou seja, x = 2^(11/6), y = 2^(5/6) e z =
> 2^(7/3).
>
> Oi Shine,
>
> eu não entendi a passagem 8xy + 2z^2 >= 4xyz. Não pode ser só
> desigualdade das médias, porque essa é homogênea, e todos os termos da
> esquerda são de ordem dois. Acho que faltou uma dica para o seu caro
> leitor.
>
> Pensando um pouco mais, eu resolveria com multiplicadores de Lagrange
> (e portanto com derivadas). Mas se fosse antes de aprender Lagrange,
> eu teria feito assim:
>
> Note que se z é fixo, temos que minimizar (x + 2y)^2, com xy =
> constante. (Aplicando a famosa técnica "escolha produtos notáveis que
> vão te ajudar".) Pela MA >= MG, obtemos x = 2y (como todo mundo
> obteve...).
>
> xy = 32/z, x = 2y => 2y^2 = 32/z => y^2 = 16/z, x^2 = 4*16/z e
> portanto x^2 + 4xy + 4y^2 = 4*16/z + 4*32/z + 4*16/z = 4*32*2/z.
>
> Queremos minimizar 4*32*2/z + 2z^2. Pela desigualdade das médias com 3
> termos: 4*32/z + 4*32/z + 2z^2 >= 3 * (4*32 * 4*32 * 2)^1/3 = 3 *
> (2^(2+5+2+5+1))^1/3 = 3 * 2^5 = 3 * 32 = 96. A igualdade ocorre para
>
> 4*32/z = 2z^2 <=> 64 = z^3, ou seja z = 4, y = 4/raiz(z) = 4/2 = 2, x = 4.
>
> Verificando: x^2 = 4^2 = 16
> 4xy = 4*2*4 = 32
> 4*y^2 = 4*2^2 = 16
> 2z^2 = 2*4^2 = 32
> Somando = 96.
>
> --
> Bernardo Freitas Paulo da Costa
>
> =
> Instruções para entrar na lista, sair da lista e usar a lista em
> http://www.mat.puc-rio.br/~obmlistas/obm-l.html
> =
>


[obm-l] Re: [obm-l] Geometria Plana - Triângulo

2012-08-28 Por tôpico Carlos Victor
Olá  Arkon ,
Uma solução é :

Seja O o ortocentro de ABC . Observe que o triângulo AOC é semelhante ao
triângulo OEH  , pois o quadrilátero ACHE é inscritível . Seja x  = EH ,
então 7/x = AO/EO e como OE = OA.cosB . Usando a lei dos cosenos  encontre
cosB = 1/5 e daí x =7/5 , ok ? .Acredito que pensar no círculo dos nove
pontos  pode também resolver .

Confira as contas .

Abraços

Carlos  Victor

Em 28 de agosto de 2012 19:31, arkon  escreveu:

> Pessoal, qual o bizu?
> Â
> Em um triângulo ABC, traçam-se as alturas AH e CE. Se AB=5m, BC=6m e
> AC=7m, calcule EH.
> Â
> (A) 7/5 m (B) 9/5 m (C) 10/7 m (D) 10/3 m (E) 2
> =
> Instruções para entrar na lista, sair da lista e usar a lista em
> http://www.mat.puc-rio.br/~obmlistas/obm-l.html=


Re: [obm-l] Divisibilidade

2012-05-20 Por tôpico Carlos Victor
Olá Thiago ,

Pense assim :

43x+75y = 38x +76y  + 5x -y

Basta então mostrar que  5x-y é múltiplo de 19 .

5x-y = 5(5x-y) - 2(3x+7y) = 19x - 19y . Como  3x+7y =19k , temos que 43x+
75y também é .

Abraços

Carlos  Victor

Em 11 de maio de 2012 08:25, Thiago Bersch escreveu:

>  Mostre que se [image: 19|3x+7y] então [image: 19|43x+75y]
>


[obm-l] Re: [obm-l] Congruência modulo n

2011-12-16 Por tôpico Carlos Victor
Olá  Kleber ,

Usando o teorema de Euler temos que  12^20 é congruo a 1 mod (25). Elevando
a 657 , temos que  12^13140 é congruo 1 mod(25).Logo , basta ver a divisão
de 12^5 por 25 , ok ?.

Teorema de Euler :Sejam a,m naturais com m > 1 e mdc(a,m) =1. Então  a^(fi
de m) é congruo a 1 modm .

Abraços

Carlos  Victor

Em 16 de dezembro de 2011 13:49, Kleber Bastos escreveu:

>
> Queria saber qual o método para calcular:
> Dado que 12^13145(mod 25), calcular o resto da divisão de 12^13145 por 25.
>
> Desde já agradeço a ajuda.
> Abraços, Kleber.
>
>
>


[obm-l] Re: [obm-l] Exercício proposto(Eureka!)

2011-07-28 Por tôpico Carlos Victor
Olá Marcone ,
Sabendo  que  : cos(pi/7) - cos(2pi/7) + cos(3pi/7) = 1/2 , use as
expressões de  cos3x  e de  cos2x  em função de cosx , com  x = pi/7 ,  ok ?

Abraços

Carlos  Victor

Em 28 de julho de 2011 18:24, marcone augusto araújo borges <
marconeborge...@hotmail.com> escreveu:

>  Prove que x = 2cos(pi/7) satisfaz a equação x^3 + x^2 -2x + 1 = 0.
>
> Há um exercício resolvido na revista,mostrando que pi/7 é raíz da equação
> 8x^3 - 4x^2 -4x + 1 = 0
> Mas não estou conseguindo e peço ajuda
> Agradeço desde já.
>
>
>
>


Re: [obm-l] Re: [obm-l] Geometria OBM

2011-07-24 Por tôpico Carlos Victor
Linda Solução
 do Julio.

Carlos Victor

2011/7/24 Julio César Saldaña 

>
>
> Uma solução geométrica:
>
> Sabemos que O2A=O2B.
> Prolongue ou estique BO1, (desculpem o protunhol) até um ponto P tal que
> O2P=O2B
> (=O2A). Calculemos uns ángulos: BPO2=20, PO2C=40, AO2P=60, então o
> triángulo
> O2AP é equilátero, ouseja PA=AO2, PAC=30 e a reta AO1 será mediatriz de
> PO2.
> Então PO1=O1O2, então O1O2P=20, finalmente  BO1O2=20+20=40
>
>
>
> Julio Saldaña
>
>
> -- Mensaje original ---
> De : obm-l@mat.puc-rio.br
> Para : obm-l@mat.puc-rio.br
> Fecha : Sun, 24 Jul 2011 19:18:18 -0300
> Asunto : Re: [obm-l] Geometria OBM
> >Olá João ,
> >
> >Houve um erro na digitação : onde está  AO1 =y  lê-se  BO1 =y . ok ? .
> >Desculpe o erro .
> >
> >Abraços
> >
> >Carlos  Victor
> >
> >Em 24 de julho de 2011 19:11, Carlos Victor  >escreveu:
> >
> >> Olá João ,
> >>
> >> Vamos inicialmente a uma solução trigonométrica :
> >>
> >> Seja z o ângulo pedido .Sejam também AB =a ; AO2 = x  e AO1= y.Então
> >> teremos :
> >>
> >> Triang  *BO1O2 : y/sen(160-z) = x/sen*z
> >>
> >> Triang ABO2 : x/sen50 = a/sen80
> >>
> >> Triang BO1A : y/sen80 = a/sen70
> >>
> >> Logo :sen(20+z) = 4cos10.sen20.senz  ; ou  sen(20+z) -senz = 2senz.sen10
> >>
> >> Donde  cos(z+10) = senz ; ou seja  z = 40 ° .
> >>
> >> Tentarei uma solução geométrica . ok ?
> >>
> >> Abraços
> >>
> >> Carlos  Victor
> >>
> >>
> >>
> >>
> >>
> >>
> >> 2011/7/24 João Maldonado 
> >>
> >>>  Inglaterra -- 1970
> >>>
> >>> No triângulo ABC, AB = AC e A=80°,  dado O1 em AC tal que O1BC =  20° e
> O2
> >>> em  BC tal que   CAO2 = 30°,  calcule   BO1O2
> >>>
> >>> Obrigado
> >>> João
> >>>
> >>
> >>
> >
>
> __
> Si desea recibir, semanalmente, el Boletín Electrónico de la PUCP, ingrese
> a:
> http://www.pucp.edu.pe/puntoedu/suscribete/
>
> =
> Instruções para entrar na lista, sair da lista e usar a lista em
> http://www.mat.puc-rio.br/~obmlistas/obm-l.html
> =
>


Re: [obm-l] Geometria OBM

2011-07-24 Por tôpico Carlos Victor
Olá João ,

Houve um erro na digitação : onde está  AO1 =y  lê-se  BO1 =y . ok ? .
Desculpe o erro .

Abraços

Carlos  Victor

Em 24 de julho de 2011 19:11, Carlos Victor escreveu:

> Olá João ,
>
> Vamos inicialmente a uma solução trigonométrica :
>
> Seja z o ângulo pedido .Sejam também AB =a ; AO2 = x  e AO1= y.Então
> teremos :
>
> Triang  *BO1O2 : y/sen(160-z) = x/sen*z
>
> Triang ABO2 : x/sen50 = a/sen80
>
> Triang BO1A : y/sen80 = a/sen70
>
> Logo :sen(20+z) = 4cos10.sen20.senz  ; ou  sen(20+z) -senz = 2senz.sen10
>
> Donde  cos(z+10) = senz ; ou seja  z = 40 ° .
>
> Tentarei uma solução geométrica . ok ?
>
> Abraços
>
> Carlos  Victor
>
>
>
>
>
>
> 2011/7/24 João Maldonado 
>
>>  Inglaterra -- 1970
>>
>> No triângulo ABC, AB = AC e A=80°,  dado O1 em AC tal que O1BC =  20° e O2
>> em  BC tal que   CAO2 = 30°,  calcule   BO1O2
>>
>> Obrigado
>> João
>>
>
>


Re: [obm-l] Geometria OBM

2011-07-24 Por tôpico Carlos Victor
Olá João ,

Vamos inicialmente a uma solução trigonométrica :

Seja z o ângulo pedido .Sejam também AB =a ; AO2 = x  e AO1= y.Então teremos
:

Triang  *BO1O2 : y/sen(160-z) = x/sen*z

Triang ABO2 : x/sen50 = a/sen80

Triang BO1A : y/sen80 = a/sen70

Logo :sen(20+z) = 4cos10.sen20.senz  ; ou  sen(20+z) -senz = 2senz.sen10

Donde  cos(z+10) = senz ; ou seja  z = 40 ° .

Tentarei uma solução geométrica . ok ?

Abraços

Carlos  Victor





2011/7/24 João Maldonado 

>  Inglaterra -- 1970
>
> No triângulo ABC, AB = AC e A=80°,  dado O1 em AC tal que O1BC =  20° e O2
> em  BC tal que   CAO2 = 30°,  calcule   BO1O2
>
> Obrigado
> João
>


Re: [obm-l] Fwd: Identidade de Euler

2011-04-28 Por tôpico Carlos Victor
Oi  Mestre  Nehab ,
Gostei da sugestão e mais ainda  das n pessoas que moram em Nilópolis (
minha  terrinha).

Abraços

Carlos  Victor

Em 28 de abril de 2011 17:21, Carlos Nehab  escreveu:

> Oi, Fábio,
>
> Não resisti:
>
> Resolva os seguinte problema de duas maneiras (uma técnica básica e útil
> para resolver identidades deste tipo).
> De quantas maneira posso formar comissões de p pessoas, a partir de um
> total de m + n pessoas, sendo m o total de pessoas que moram no Maracanã e n
> as pessoas que moram em Nilópolis?
>
> Abraços,
> Nehab
>
> Em 28/4/2011 13:24, fabio henrique teixeira de souza escreveu:
>
> -- Mensagem encaminhada --
>> De: fabio henrique teixeira de souza
>> Data: 28 de abril de 2011 08:52
>> Assunto: Identidade de Euler
>> Para: obm-l@mat.puc-rio.br
>>
>>
>> Pessoal, estou batendo cabeça e não consigo demonstrar que
>> C(m,0).C(n,p) + C(m,1).C(n,p-1) + C(m,2).C(n,p-2) + ... + C(m,p).C(n,0) =
>> C(m+n,p)
>>
>> Alguém pode me dar uma dica?
>>
>>
> =
> Instruções para entrar na lista, sair da lista e usar a lista em
> http://www.mat.puc-rio.br/~obmlistas/obm-l.html
> =
>


Re: [obm-l] distancia no trapezio

2011-01-09 Por tôpico Carlos Victor
Olá  Luis,

Trace de D uma paralela ao lado BC que encontra AB por exemplo em E. Observe
que BE = 4 e o triângulo EDA é isósceles  e, donde EA = 6. Logo AB = 10 , ok
? . Verifique se esta ideia está correta .

Abraços

Carlos Victor

Em 9 de janeiro de 2011 13:56, Luís Lopes  escreveu:

>  Sauda¸c~oes,
>
> Pediram-me para resolver o seguinte problema.
>
> No trapezio ABCD, CD=4 e DA=6. E tambem B=\alpha e D=2\alpha.
> Calcular AB.
>
> O "desenho" do trapezio eh
>
>   C o---o D
>
>
>
> B o---o A
>
>
> Abra¸cos,
> Luis
>
>


RE: [obm-l] Ajuda URGENTE

2007-01-22 Por tôpico Carlos Victor


Olá  ,
Trace  a  altura  AH  relativa  ao  lado  BC . Escolha  um 
ponto  Q  sobre  AH , tal  que  o  triângulo BCQ  seja  equilátero . 
Agora  , verifique  a  congruência  dos  triângulos  ABQ  e 
BPA   e,  daí  conclua  que  o  o ângulo  BQA = 150°  e consequentemente  o 
ângulo  pedido   será  30°, ok ?


[]´s  Carlos  Victor







At 16:56 22/1/2007, Filipe de Carvalho Hasché wrote:

Em um triângulo ABC, tem-se que os ângulos ABC = ACB = 80º. Se P é um ponto
sobre o lado AB tal que AP = BC, a medida do ângulo BPC é igual a:


=

Eu emperrei tb, heheheh
Mas fiz no Cabri e deu 30°.
E nem assim consegui desvendar..

Abraços,
FC.

_
MSN Busca: fácil, rápido, direto ao ponto.  http://search.msn.com.br

=
Instruções para entrar na lista, sair da lista e usar a lista em
http://www.mat.puc-rio.br/~nicolau/olimp/obm-l.html
=




=
Instruções para entrar na lista, sair da lista e usar a lista em
http://www.mat.puc-rio.br/~nicolau/olimp/obm-l.html
=


Re: [obm-l] IME-65/66

2007-01-20 Por tôpico Carlos Victor


Olá  Arkon ,
observe  o seguinte :

Depois  de  provar   que  a série é  convergente ,faça  o seguinte :

S =lim1/4 [1/1.3 - 1/3.5  + 1/3.5 - 1/5.7  +... -  1/(2n+1)(2n+3) ] = 
lim1/4[ 1/3 - 1/(2n+1)(2n+3) ]  quando  n  cresce   , ou  seja  S = 1/12  , 
ok ?


Confira  as contas .

[]´s  Carlos Victor


At 11:59 20/1/2007, arkon wrote:

POR FAVOR, QUAL O MACETE PARA RESOLVER ESTA QUESTÃO?
(IME-65/66)

CALCULE A SOMA DA SÉRIE:

1/1.3.5 + 1/3.5.7 + 1/5.7.9 + ...

DESDE JÁ AGRADEÇO A TODOS QUE ESTÃO ME AJUDANDO.

ABRAÇOS.




=
Instruções para entrar na lista, sair da lista e usar a lista em
http://www.mat.puc-rio.br/~nicolau/olimp/obm-l.html
=


Re: Res: [obm-l] Inducao

2007-01-17 Por tôpico Carlos Victor


Olá  Klaus ,

Observe o seguinte : inicialmente eu estou vendo um quadrado , quando o 
dividimos  em quatro partes , ficam  4 partes  menores , ok ? ; ou seja , 
eu tinha  um quadrado  e  agora  eu tenho 4 quadradinhos . Daí 
acrescentei  3  , ok ?


[]´s   Carlos  Victor




At 16:55 17/1/2007, Klaus Ferraz wrote:

Ola Carlos
 Nao entendi. Porque quando dividimos um quadrado em 
4 partes estamos acrescentando 3 quadradinhos ao quadrado original


-,




=
Instruções para entrar na lista, sair da lista e usar a lista em
http://www.mat.puc-rio.br/~nicolau/olimp/obm-l.html
=


Re: [obm-l] Inducao

2007-01-17 Por tôpico Carlos Victor


Olá  Luís ,
Divida  inicialmente  o quadrado  original em  16  quadradinhos  , 
apague  os  9  quadradinhos  do  canto  superior  esquerdo 
(por  exemplo)  , ficará  um  quadrado  maior  , junto  com 
os  outros   7   quadradinhos  que  sobraram , ok  ?


Abraços   e  satisfação em falar  com você

[]´s  Carlos  Victor






At 12:17 17/1/2007, Luís Lopes wrote:

Sauda,c~oes,

Oi Carlos Victor,

Como obter 8 quadrados?

Seguindo suas idéias dividi o quadrado inicial em
9 e 16 quadrados iguais. Com os 9 quadrados
gera-se a seqüência 6,9,12, E com os 16,
a seqüência 4,7,10,13,16,19...

A 8,11,14,... não consegui. Talvez se eu soubesse
resolver 3k+8=n^2 ajudasse.

[]'s
Luís


From: Carlos  Victor <[EMAIL PROTECTED]>
Reply-To: obm-l@mat.puc-rio.br
To: obm-l@mat.puc-rio.br, obm-l@mat.puc-rio.br
Subject: Re: [obm-l] Inducao
Date: Tue, 16 Jan 2007 19:58:49 -0200


Olá  Klaus, para  o segundo :

Observe  que  quando dividimos  um quadrado  em 4 partes , na verdade
acrescentamos  3 quadradinhos  ao  quadrado original . Pensando  desta
forma basta você  conseguir  dividir  um  quadrado   em 6 , 7 e 8  outros
quadradinhos, pois  a partir  desses  usa  o procedimento inicial . 
Com  um pouco  de paciência  verifica-se  que   dividir  um 
quadrado  em  6 , 7  e

8  outros  quadradinhos   não é  difícil  e ,  consequentemente  teremos
as seguintes   sequências  :

1) 6 ,9 , 12 , ...

2) 7 , 10 , 13 , ...

3) 8 , 11 , 14 , ...

Unindo  as sequências  temos  os  naturais   a partir  de  6 , ok ?


[]´s  Carlos  Victor




At 18:27 16/1/2007, Klaus Ferraz wrote:
1)Prove que todo inteiro positivo pode ser escrito como potencias de 2 
com expoentes distintos

2)Prove que um quadrado pode ser dividido em n quadrados para n>=6.
3)Prove que [1.3.5..(2n-1)]/[2.4.6.8...2n]=<1/sqrt(2n+1)

Grato.


_
MSN Busca: fácil, rápido, direto ao ponto.  http://search.msn.com.br

=
Instruções para entrar na lista, sair da lista e usar a lista em
http://www.mat.puc-rio.br/~nicolau/olimp/obm-l.html
=




=
Instruções para entrar na lista, sair da lista e usar a lista em
http://www.mat.puc-rio.br/~nicolau/olimp/obm-l.html
=


Re: [obm-l] Inducao

2007-01-16 Por tôpico Carlos Victor


Olá  Klaus, para  o segundo :

Observe  que  quando dividimos  um quadrado  em 4 partes , na 
verdade  acrescentamos  3 quadradinhos  ao  quadrado original . 
Pensando  desta  forma basta você  conseguir  dividir  um  quadrado   em 6 
, 7 e 8  outros  quadradinhos, pois  a partir  desses  usa  o procedimento 
inicial . Com  um pouco  de paciência  verifica-se  que   dividir  um 
quadrado  em  6 , 7  e  8  outros  quadradinhos   não é  difícil  e 
,  consequentemente  teremos  as seguintes   sequências  :


1) 6 ,9 , 12 , ...

2) 7 , 10 , 13 , ...

3) 8 , 11 , 14 , ...

Unindo  as sequências  temos  os  naturais   a partir  de  6 , ok ?


[]´s  Carlos  Victor








At 18:27 16/1/2007, Klaus Ferraz wrote:
1)Prove que todo inteiro positivo pode ser escrito como potencias de 2 com 
expoentes distintos

2)Prove que um quadrado pode ser dividido em n quadrados para n>=6.
3)Prove que [1.3.5..(2n-1)]/[2.4.6.8...2n]=<1/sqrt(2n+1)

Grato.


__
Fale com seus amigos de graça com o novo Yahoo! Messenger
http://br.messenger.yahoo.com/




=
Instruções para entrar na lista, sair da lista e usar a lista em
http://www.mat.puc-rio.br/~nicolau/olimp/obm-l.html
=


Re: [obm-l] PG

2006-12-29 Por tôpico Carlos Victor


Olá   Marcus ,

Faça o  seguinte : suponha  que  as ordens de  1, 2  e 5  sejam 
respectivamente  m , p  e  n . Use  a expressão  do termo geral  com 
razão  igual  a q  e  conclua  que  :


5^(p-m) = 2^(n-m)  e já  que   m,n e p são  naturais  , teremos p=m=n 
.  Conclusão : 1,2e5  não podem  pertencer  a uma  mesma  PG , ok ? 
confira  as contas.


[]´s  Carlos  Victor




At 10:06 29/12/2006, Marcus Aurélio wrote:

Podem os números 1, 2 e 5 pertencer à mesma PG



=
Instruções para entrar na lista, sair da lista e usar a lista em
http://www.mat.puc-rio.br/~nicolau/olimp/obm-l.html
=



=
Instruções para entrar na lista, sair da lista e usar a lista em
http://www.mat.puc-rio.br/~nicolau/olimp/obm-l.html
=


Re: [obm-l] probleminha da en

2006-12-11 Por tôpico Carlos Victor

Olá  Arkon,

Como  dizia  o nosso mestre MORGADO ,  um " truque"   para  este tipo  de 
problema é :


Como  são  quatro  conjuntos , o que  ultrapassar  a  300%  será  a 
quantidade   da interseção  dos  conjuntos . Se  tivermos  n  conjuntos , o 
que  ultrapasar  a (n-1)x100%   será  o mínimo  da interseção , ( tente 
provar) ,ok ?


[]´s   Carlos  Victor




At 16:58 11/12/2006, arkon wrote:
Gostaria que alguém da lista me enviasse a resolução de mais uma questão 
da en, por favor:


grato.

Se 70% da população gostam de samba, 75% de choro, 80% de bolero e 85% de 
rock, quantos por cento da população, no mínimo, gostam de samba, choro, 
bolero e rock?


a) 5%.
b) 10%.
c) 20%.
d) 45%.
e) 70%.

Obs.: A alternativa correta é a letra b.



=
Instruções para entrar na lista, sair da lista e usar a lista em
http://www.mat.puc-rio.br/~nicolau/olimp/obm-l.html
=


Re: [obm-l] ajuda em tres questoes

2006-12-06 Por tôpico Carlos Victor


Olá  Fábio ,

1) Para  a primeira  questão  faça  o seguinte : agrupar  onde tiver  o 
parâmetro  " a"  e  depois  anular  o fator  que estiver  multiplicando  o 
parâmetro . Neste  momento  você irá  encontrar  uma equação do seg 
grau  em  x  e  consequentemente  encontrará  dois  pontos  no plano  e 
daí  achar  a distância  entre eles, ok ?


2)  Para  o segundo : trace  a bissetriz  do ângulo  dobrado  e  utilize  o 
teorema  da bissetriz  interna junto  com uma semelhança  que irá  aparecer 
, ok ?


3) Para o terceiro : faça  duas  semelhanças   e utilize  o fato  de que  o 
raio do circulo  inscrito é dado por " p-a" , onde p é  o semi-perímetro  e 
a é a hipotenusa , ok ?


[]´s   Carlos  Victor








At 20:20 5/12/2006, Fabio Silva wrote:

Quem puder dê uma ajuda estou estudando para futuros
concursos:

Considerem-se as funções quadráticas definidas por
y=(a+1)x^2 ­ 2ax ­ (3a +7), variável x e parâmetro a.
Todos os gráficos apresentam uma corda comum. Qual
comprimento da corda? Resp: 4sqrt5.


Considerem-se um triangulo ABC onde a medida do ângulo
A é o dobro da medida de B. A medida do lado a, oposto
ao ângulo A, em função dos lados b e c, é_.
Resp: sqrt (b^2 + bc)

Considere um triangulo ret de hip a, sendo h a altura
relativa a hip e r o raio do circulo inscrito no
triangulo. Inscrevem-se neste triangulo um quadrado de
lados sobre os catetos e vértice na hip, e um outro de
lado sobre a hip e vértices sobre os catetos. A razão
entre as medidas dos lados do primeiro e do segundo
quadrado é:. Resp: a+r sobre a+2r


Vlw amigos matemáticos!!!





Yahoo! Music Unlimited
Access over 1 million songs.
http://music.yahoo.com/unlimited
=
Instruções para entrar na lista, sair da lista e usar a lista em
http://www.mat.puc-rio.br/~nicolau/olimp/obm-l.html
=




=
Instruções para entrar na lista, sair da lista e usar a lista em
http://www.mat.puc-rio.br/~nicolau/olimp/obm-l.html
=


Re: [obm-l] Fatoração

2006-10-16 Por tôpico Carlos Victor


Olá  Bruna ,
Adicione  e  subtraia  os  fatores : x^10 ,x^9 , ... x  .Depois  é só 
agrupar  os  fatores : x^11+x^10+x^9 , -(x^10+x^9+x^8)  e  assim 
por  diante  ; onde   o fator  x^2+x+1  será  comum . Conclusão



x^11 + x^7 + 1 = ( x^2+x+1)(x^9-x^8+x^6-x^4+x^3-x+1) , ok ?

[]´s  Carlos  Victor



At 14:05 14/10/2006, Bruna Carvalho wrote:

Fatorar:

P(x) = x^11 + x^7 + 1.




=
Instruções para entrar na lista, sair da lista e usar a lista em
http://www.mat.puc-rio.br/~nicolau/olimp/obm-l.html
=


Re: [obm-l] trigonometria - O retorno!

2006-10-07 Por tôpico Carlos Victor

Olá  marinho ,
Faça o seguinte :

A/2+B/2+C/2 = 90°  , então  cos( A/2+B/2) = sen(C/2) -->  cos(A/2).Cos(B/2) 
- sen(A/2).sen(B/2) = sen(C/2) ;


multiplique os dois  lados por 4sen(A/2).sen(B/2)  , adicione  uma unidade 
ambos os membros ,


 substitua senA =2sen(A/2).cos(B/2) ,senB=2sen(B/2).cos(B/2) 
e   sen(A/2)^2 =(1-cosA)/2 , fazendo o mesmo  para sen(B/2)^2 .


No  final teremos :  senA.senB -+cosA+cosB -cosA.cosB = 1 + 
4sen(A/2).sen(B/2).sen(C/2)  e ,


como senA.senB-cosA.cosB=cosC   chegaremos  ao resultado  pedido , ok ?

[]´s  Carlos  Victor





At 20:05 7/10/2006, Marinho Kamiroski wrote:

Essa é meio q clássica, vem me perseguindo a meses e nada de sair.

cos(a) + cos(b) + cos(c) = 1 + 4*sen(a/2)*sen(b/2)*sen(c/2)
com a + b + c =180° (angulos internos de um triangulo)

_
Descubra aqui como mandar Torpedos Messenger! 
http://www.msn.com.br/artigos/maguire/default.asp 
http://www.msn.com.br/artigos/maguire/default.asp


=
Instruções para entrar na lista, sair da lista e usar a lista em
http://www.mat.puc-rio.br/~nicolau/olimp/obm-l.html
=



=
Instruções para entrar na lista, sair da lista e usar a lista em
http://www.mat.puc-rio.br/~nicolau/olimp/obm-l.html
=


Re: [obm-l] LIMITES

2006-05-21 Por tôpico Carlos Victor


Olá ,
Para o segundo limite temos  :
lim(x-->+inf) sen(x^1000)/x  =  lim(
1/x.sen(x^1000)   , como sendo  uma  função 
infitesima multiplicada  por  um  limitada ; ou 
seja   a resposta  é  zero .
Tem  certeza  que  a  questão   (1) 
esta  correta  ?
[]´s  Carlos  Victor

At 10:37 21/5/2006, Klaus Ferraz wrote:
1)Determine lim(n->+inf)
(1+1/2)*(1+1/2^2)*(1+1/2^3)*...*(1+1/2^n).
2)Determine lim(x-->+inf) sen(x^1000)/x
 
Grato.

Yahoo!
Messenger com voz - Instale agora e faça ligações de graça.




Re: [obm-l] Geometria plana

2006-01-26 Por tôpico Carlos Victor



Olá  Vinícius ,
Sejam  d1, d2 e d3  as distâncias e L  o lado  do
triângulo ;escolha  um ponto   exterior  ao
triângulo  de tal  maneira  a construir  um 
triângulo  equilátero de lados iguais  a  d1,  por
exemplo . Utilize  a  congruência   de 
triângulos( triângulos  de lados  L ,d1 e d3)  e a 
Lei  do  co-seno( como o Júnior  comentou  em
um  dos  seus e-mails)   para 
chegar   à  solução , ok ?
[]´s  Carlos  Victor

At 17:40 25/1/2006, vinicius aleixo wrote:
Como posso determinar a área de
um triagulo equilátero conhecendo a distancia de um ponto qualquer (P) em
seu interior aos vértices do triângulo(a,b,c)??
 
Abraços,
 
Vinícius Meireles Aleixo

Yahoo! doce lar.
Faça
do Yahoo! sua homepage. 



Re: [obm-l] Teoria dos Numeros

2006-01-26 Por tôpico Carlos Victor


Olá  Klauss ,
(x+1)^3 - x^3 = y^2   , onde  3(2x+1)^2 = (2y-1)(2y+1) .
Observe  que  podemos  concluir  que  
:
a) Ou  2y-1 = a^2    e  2y+1 = 3b^2 

b) Ou  2y-1 = 3c^2    e  2y+1 =
d^2    .

Observe  que 3b^2 =  a^2 
+2   é  a única  que  pode  ocorrer 
e,  como    a é  ímpar   ,
podemos  escrever  
a = 2t +1   e   4y = 2(a^2+1)  
implicando    y =   t^2  + (t+1)^2  ,
ok  ?
 OBS : (1) Esta  questão  se  encontra  no 
Livro   POWER  PLAY    de  
EDWARD J.  BARBEAU  da  MAA ; inclusive  com 
a  solução  acima 
(2)  O  interessante  é  que  para  3x^2+3x
+1 =y^2   tem  para  solução  geral  
:
x1 = 4y+7x+3   e  y1  =  7y+12x+6 
com   x  e  y conhecidos  . Exemplo 
:  x1 = 104  e y1  =181 ; Lindo  não  é 
?

[]´s   Carlos  Victor


At 20:23 24/1/2006, Klaus Ferraz wrote:
Mostre que a diferença entre os
cubos de dois numeros inteiros consecutivos é igual ao quadrado de um
inteiro, entao esse inteiro é igual a soma dos quadrados de dois inteiros
consecutivos.
Ex: 8^3-7^3=169.   2^2+3^2=13.
 
Grato.

Yahoo! doce lar.
Faça
do Yahoo! sua homepage. 



Re: [obm-l] MAXIMO

2006-01-07 Por tôpico Carlos Victor



Olá  ,
Use  o multiplicador  de Lagrange  para  a
função   F = E -  k( a+b+c-1=0)   , encontre 
a=b=c=1/3   e  que  o valor  máximo 
de  
E  é igual  a  6sqrt(2) , ok   ?
[ ]´s  Carlos  Victor

At 10:36 7/1/2006, Klaus Ferraz wrote:
Calcule o valor maximo da
expressao E=raiz(9a+5)+raiz(9b+5)+raiz(9c+5), a, b e c reais positivos.,
a+b+c=1

Yahoo! doce lar.
Faça
do Yahoo! sua homepage. 



[obm-l] Re: [obm-l] 3 Questõeszinahs de Calculo + 1 de bonus pra se pensar!

2006-01-06 Por tôpico Carlos Victor


Olá,
Para  a primeira  questão  ítem (b)  é  só  lembrar  do seguinte : Seja 
f  uma  função  contínua  em [0,1] , 
lim{ 1/n[f(1/n) + f(2/n) +... f(n/n)]} = integral  de f(x)dx   
com  0< x < 1   e  no  problema   a  função
é  
f(x) =  sqrt(1/(1+x)) . Determinando  a integral definida encontraremos 
o resultado  em  questão , ok ?

Para  a Questão  Especial  você  encontrará   uma  solução  elegante
( usando  o princípio de Cavalieri ) no  Livro   de  Olimpíadas 
Brasileiras  de Matemática (  1ª  a  8ª)  da  SBM .  A resposta  
é   16r^3 /3 .

Para  a questão  1a  basta  usar  o mesmo  fato  de  1b (  de outra 
mensagem ) . Para  a  segunda  questão , use  o fato de que  a função  é 
crescente  em [0,1] e  considere o  quadrado  de lado  1  com  um dos
vértices  na origem . observe  que  1/3  é  valor  da  área  sob  a curva
 e , que  a parte  acima da  curva  e dentro  do quadrado é  2/3 , que 
será   o valor da área  sob  a curva quando  considerarmos  a função
 inversa , ok ?( já  que  a função inversa  é simétrica  em relação à 
reta y=x)

[]´s  Carlos  Victor



At 23:13 4/1/2006, you wrote:
Olá
amigos...
Estou com uma lista enorme de exercícios de calculo aqui comigo e fiquei
enrolado em 3 dessas questões...
aí vão elas:
 
 
 



[obm-l] Re: [obm-l] 3 Questõeszinahs de Calculo + 1 de bonus pra se pensar!

2006-01-05 Por tôpico Carlos Victor



Olá ,
Para  a questão  1a  basta  usar  o mesmo 
fato  de  1b (  de outra  mensagem ) . Para 
a  segunda  questão , use  o fato de que  a
função  é  crescente  em [0,1] e  considere o 
quadrado  de lado  1  com  um dos vértices  na
origem . observe  que  1/3  é  valor  da 
área  sob  a curva  e , que  a parte  acima
da  curva  e dentro  do quadrado é  2/3 , que 
será   o valor da área  sob  a curva quando 
considerarmos  a função  inversa , ok ?( já  que  a
função inversa  é simétrica  em relação à  reta
y=x)
[ ]´s  Carlos  Victor


At 23:13 4/1/2006, João Vitor wrote:
Olá
amigos...
Estou com uma lista enorme de exercícios de calculo aqui comigo e fiquei
enrolado em 3 dessas questões...
aí vão elas:
 
 
 



[obm-l] Re: [obm-l] 3 Questõeszinahs de Calculo + 1 de bonus pra se pensar!

2006-01-05 Por tôpico Carlos Victor


Olá  ,
Para  a primeira  questão  ítem (b)  é  só 
lembrar  do seguinte : Seja  f  uma  função 
contínua  em [0,1] , 
lim{ 1/n[f(1/n) + f(2/n) +... f(n/n)]} = integral  de
f(x)dx    com  0< x <
1   e  no  problema   a  função
é  
f(x) =  sqrt(1/(1+x)) . Determinando  a integral definida 
encontraremos  o resultado  em  questão , ok ?

Para  a Questão  Especial  você 
encontrará   uma  solução  elegante ( usando  o
princípio de Cavalieri ) no  Livro   de 
Olimpíadas  Brasileiras  de Matemática (  1ª  a 
8ª)  da  SBM .  A resposta   é   16r^3
/3 .

[]´s  Carlos  Victor


At 23:13 4/1/2006, João Vitor wrote:
Olá
amigos...
Estou com uma lista enorme de exercícios de calculo aqui comigo e fiquei
enrolado em 3 dessas questões...
aí vão elas:
 
 



Re: [obm-l] Maple

2006-01-04 Por tôpico Carlos Victor



Olá  Teresa,
Você   deve  usar  o símbolo   " 
asterisco"  para  a multiplicação :
plot(x^2+5*x+6,x=-20..20);  ok  ?
Isto   resolverá  .
[]´s  Carlos  victor
At 19:24 4/1/2006, Maria Teresa wrote:
Estou usando
o Maple pela primeira vez e queria ver o gráfico de uma função. Digitei
assim:
 
plot(x^2+5x+6,x=-20..20);
 
e não funcionou. Tem a mensagem
 
Error, missing operator or ";"
mas como tem ; então, peço ajuda para saber o que está faltando.
Obrigada,
Maria Teresa



Re: [obm-l] 2 probleminhas

2006-01-04 Por tôpico Carlos Victor

Olá  Fábio ,

Talvez  o enunciado  da primeira questão  seja :

Em  um grupo  de 20  pessoas qual a probabilidade de
que haja pelo menos 2 delas nascidas num mesmo mês  e no  mesmo  dia ?( 
ou  seja , aniversário  no mesmo  dia)


Esta  questão  está  resolvida   no Matemática  Elementar  (  vol 5)  e  no 
Livro  do  Morgado (  Análise  Combinatória   e  probabilidade da  SBM) . 
A  resposta  é


aproximadamente  41% , Ok ?  Verifique

[]´s  Carlos  Victor




At 23:38 3/1/2006, Fabio Silva wrote:

Quem quiser ajudar-me será bem vindo:
1) Em um grupo de 20 pessoas, qual a probabilidade de
que haja pelo menos 2 delas nascidas num mesmo mês?

2) Ao preço de p reais, um fabricante consegue vender,
diariamente, 800-200p pacotes de biscoito. A
fabricação diária de x pacotes de biscoito custa-lhe
100+0,2x reais. Qual é o valor de p para o qual o
lucro do fabricante é máximo?

Desde já agradeço a ajuda dos colegas.



__
Yahoo! DSL ­ Something to write home about.
Just $16.99/mo. or less.
dsl.yahoo.com

=
Instruções para entrar na lista, sair da lista e usar a lista em
http://www.mat.puc-rio.br/~nicolau/olimp/obm-l.html
=




=
Instruções para entrar na lista, sair da lista e usar a lista em
http://www.mat.puc-rio.br/~nicolau/olimp/obm-l.html
=


Re: [obm-l] Geometria

2005-07-31 Por tôpico Carlos Victor

Olá  Felipe  ,( bonita  questão )

Faça  o  seguinte  : Sejam  ABC  o  triângulo e  H  o  ortocentro 
.Trace   a  mediana  AM , a  altura  AP . 
De  O  trace  uma   paralela  ao  lado  BC  e  seja  S  o  encontro  desta 
com  AP.Como  o  ortocentro , o  baricentro  e 
o  circuncentro  estão  alinhados,  se  tomarmos  x = OG  , teremos  GH = 
2x . Observe  também  que  AM = RcosA , AH = 2RcosA , PC = bcosC , AP 
=bsenC  e  AS = a/2 - PC . Agora  , considere  o triângulo  retângulo 
OSH   ,  aplique  pitágoras , e  com  as leis  dos  seno  e  dos  coseno 
em  ABC  ,  você  irá  encontrar  o  pedido  na  questão ;ok ?


Caso  não  consiga  ,  serei  mais  claro .

[]´s  Carlos  Victor





At 15:50 30/7/2005, Felipe Takiyama wrote:


Alguém poderia me ajudar com este?

Sejam, num triângulo ABC: O, o centro da circunferência circunscrita; G, o 
ponto

de intersecção das medianas; a,b e c, os lados; e R, o raio da circunferência
circunscrita. Demonstrar que:

  OG^2 = R^2 - (a^2 + b^2 + c^2)/9

Felipe

___
Navegue e Ganhe vale-presentes no Submarino.
Inscreva-se agora na promoção Mergulhou Ganhou!
www.click21.com.br/mergulhouganhou

=
Instruções para entrar na lista, sair da lista e usar a lista em
http://www.mat.puc-rio.br/~nicolau/olimp/obm-l.html
=




=
Instruções para entrar na lista, sair da lista e usar a lista em
http://www.mat.puc-rio.br/~nicolau/olimp/obm-l.html
=


Re: [obm-l] Usando o Maple

2005-07-24 Por tôpico Carlos Victor


Olá  Marcos,

Esta  substituição   não  resolveu ;  caso  exista  outra me informe , ok ?

[]´s  Carlos Victor

At 01:36 24/7/2005, Marcos Paulo wrote:

Carlos Victor wrote:


Olá  pessoal ,

Gostaria  de saber  como calcular  diretamente  usando o  Maplea 
seguinte  soma  :


> *(5+2*sqrt(13))^(1/3) + (5-2*sqrt(13))^(1/3)

*Agradeço  desde já  qualquer  ajuda


[]´s    Carlos  Victor


Tente substituir o 13 por 13.0

[]'s MP
=
Instruções para entrar na lista, sair da lista e usar a lista em
http://www.mat.puc-rio.br/~nicolau/olimp/obm-l.html
=




=
Instruções para entrar na lista, sair da lista e usar a lista em
http://www.mat.puc-rio.br/~nicolau/olimp/obm-l.html
=


  1   2   >